ATI RN Nursing Care of Children Proctored 2019 Exam Level 3 Peds Questions and Answers (Verified Answers)

What is a dictorial or authoritarian parenting style?
parents try to control the child’s behaviors and attitudes through unquestioned rules and expectations

What is an authoriatitive parenting style?
also known as democratic, parents direct the child’s behavior by setting rules and explaining the reson for each rule setting

What is passive parenting?
parents are uninvolved, indifferent, and emotionally removed

A nurse manager on a pediatric floor is preparing an education program on working with families for a group of newly hired nurses. Which of the following should the nurse include when discussing the developmental theory?
A. describes that stress is inevitable
B. emphasizes that change with one member affects the entire family
C. provides guidance to assist families adapting to stress
D. Defines consistencies in how families change
D

A nurse is assisting a group of parents of adolescents to develop skills that will improve communication. The nurse heads one parent states “my son knows he better do what I say”. Which of the parenting styles is he exhibiting?
A. Authoritarian
B. Permissive
C. Authroitative
D. Passive
A

A nurse is performimg family assessment. Which of the following should the nurse include? (select all that apply)
A. medical history
B. parents’ education level
C. child’s physical growth
D. Support systems
E. Stressors
A, B, D, E

What is the expected pulse rate of a newborn?
80 to 180/min

What is the expected pulse of a baby 1 week to 3 months?
12 to 180/min

What is the expected pulse of a child 3 months to 2 years?
70 to 150/min

What is the expected pulse of a child 2 to 10 years?
60 to 110/min

What is the expected pulse of a child 10 years and older?
50 to 90/min

What are the expected respirations fo a newborn to one year?
30 to 35/min

What are the expected respirations of a 1 to 2 year old?
25 to 30/min

What are the expected respirations of a 2 to 6 year old?
21 to 25/min

What are the expected respirations of a child 6 to 12 years old?
19 to 21/min

What are the expected respirations of a 12 year old and older?
16 to 19/min

What are the normal vitals of an infant?
HR: 80-180
RR: 30-35
BP: 65-80/40-50

Fontanels
should be flat and soft, posterior closes between 6 and 8 weeks, anterior closes between 12 and 18 months

Teeth
6 to 8 teeth by 1 year of age, 20 baby teeth and 32 permanent teeth

How long is the Moro reflex present?
until 4 months of age

How long is the Tonic neck reflex present?
until 3 to 4 months of age

How long does the Babinski reflex last?
usually until a year

Expected findings of the olfactory (I) nerve in infants , children, and adolescents
Infants: difficult to test
Children and Adolescents: indentifies smell through each nostril individually

Expected findings of optic nerve (II)?
Infants: looks at face and tracks with eyes

Children and adolescents: has intact visual acuity, peripheral vision, and color vision

Expected findings for trigeminal nerve?
infants: has rooting and sucking relfex
children and adolescents: is able to clencg teeth together and can detect touch on face with eyes closed

A nurse is preparing to assess a preschool-age child. Which of the following is an appripirate action by the nurse to prepare the child?
A. Allow the child to role play using miniature equipment
B. use medical terminology to describe what will happen
C. separate th child from her parents during examination
D. keep medical equipment visible to the child
A

A nurse is checking the vital signs of a 3-year-old during a well child visit, which of the following findings should the nurse report to the provider?
A. temperature 37.2C (99.0F)
B. Heart rate of 106/min
C. Respirations 30/min
D. Blood pressure 88/54 mmHg
C

A nurse is assessing a child’s ears. Which of the following is an expected finding?
A. Light reflex is located at the 2 o clock position
B. Tympanic membrane is red in color
C. bone landmarks are not visible
D. Cerumen is present bilaterally
D

A nurse is assessing a 6-month-old infant. Which of the following reflexes shoudl the infant exhibit?
A. Moro
B. Plantar grasp
C. Stepping
D. Tonic necl
B

A nurse is performing a neurological assessment on an adolescent. Which of the following is an appropriate reaction by the adolescent when the nurse checks the trigeminal cranial nerve? (select all that apply)
A. clencing the teeth together tightly
B. recognizing a sour tast
C. identifying smells through each nostril
D. detecing facial touches when eyes closed
E. Looking down and in with the eyes
A, D

What happens to a baby’s birth weight?
it should double by 6 months and triple by 1 year

How do infants grow?
1 inch per month (2.5cm) for 6 months, then by 12 months, height/length should be doubled

When do the first teeth arupt?
between 6 and 10 months

Gross and fine motor by 3 months
only have slight head lag

Gross and fine motor by 4 months
should be able to roll from back to side

Gross and fine motor by 5 months
should be able to roll from front to back

Gross and fine motor by 6 months
should be able to roll from back to fron and hold a bottle

Gross and fine motor by 7 months
move object from hand to hand

Gross and fine motor by 8 months
sit unsupported

Gross and fine motor by 9 months
crude pincer grasp

Gross and fine motor by 10 months
prone to sitting positiion and grasp a ratty by the handle

Gross and fine motor by 11 months
puts objects into a container and have a neater pincer grasp

Gross and fine motor by 12 months
tries to build a 2 block tower and won’t succeed

What Piaget congitive development stafe are infants in?
Sensorimotor stage, birth to 24 months, separation, object permanence around 9 months, mental representation

How many words should the infant know?
3-5 words and has concept of numbers by 1 year

What is the Erikson’s stage of development for infants?
Trust vs. Mistrust, birth to 1 year, caretake meeteing the needs of the infant

When does separation anxiety begin to occur?
between 4 and 8 months

When is there stranger fear in infants?
6 to 8 months

What toys should be used for an infant?
rattles, blocks, brightly colored toys, mirrors, patty cake

Infant Immunizations
Birth: hep B
2 months: hep B, IPV, RV, PCV, dtap, HIB
4 months: all of the 2m, hep B
6 month: all the previous
Flu shots: 6m to 1 year

Infant Nutrition
breast milk first 6 months, solids 4 to 6 months and first solid is usually iron fortified rice ceral, no juice or water is needed for first year, foods introduced one at a time over 4-7 day period to monitor for allergies

What are infant safety concers?
choking/aspiration (grapes, coins, candy)
burns (sunscnreen, handles turned away from stove, electrical outlets are covered), drowning, rear facing care seat until 2 years, crib slats are no more than 6cm, no pillows, and sleep on back

A nurse is assessing a 12 months old infant during a well-child visit. Which of the following findings should the nurse report to the provider?
A. closed anterior fontanel
B. eruption of 6 teeth
C. Birth weight doubled
D. Birth length increased by 50 %
C

A nurse is performing a developmental screening of a 10-month old infant. Which of the following fine motor skills should the nurse expect to find? (select all that apply)
A. grasp a raddle by the handle
B. try building a two-block tower
C. use a crude pincer grasp
D. Place objects into a container
E. Walkes with one hand held
A, C

A nurse is conducting a well-baby visit with a 4 motnh old infant. Which of the following immunizations should the nurse plan to administer? (select all that apply)
A. MMR
B. IPV
C. PCV
D. varicella
E. RV
B, C, E

A nurse is providing education about introducing new foods to the parents of a 4 months old infant. The nurse should recommend that the parents introduce which of the following foods first?
A. Strained yellow vegetables
B. Iron fortified cereals
C. Pureed foods
D. Whole Milk
B

A nurse is providing teaching about dental care and teething to the parent of a 9-month-old. Which of the following statements by the parent indicates an understanding of the teaching?
A. I can give my baby a warm teething ring to relieve discomfort
B. I should clean my baby’s teeth which a cool, wet washcloth
C. I can give Advil for up to 5 days while my baby is teething
D. I should place diluted juice in the bottle my baby drinks while falling asleep
B

Weight gain of todder
4x their birth weight by 30 months

Height gain of toddler
3 inches per year (7.5cm)

Head and chest growth of toddler
head and chest circumference are abotu equal, compared to when they are born, and their head is wider than their chest

gross and fine motor skills of a 15-month toddler
expect walking without help, should be able to build a 2 block tower

gross and fine motor skills of 18-month toddler
can throw a ball over head

gross and fine motor skills of 2 year old toddler
can walk up and down stairs by placing feet on each step and build a 6-7 block tower

gross and fine motor skills of 2.5 year old toddler
can jump with both feet and draw circles

Language of a toddler
1 year = 1 word senteces/hollow phrases
2 year = 2-3 word senteces

Erikson’s stage of Toddlers
autonomy vs shame and doubt, independence, begin to express selves by saying no a lot, thrive on rituals, maintian routines

Toddler appropirate activities
blocks, push pull, thick crayons, puzzle

Bathroom needs of toddlers
toilet training begins when they have recognized the sensation that they need to go potty

Immunizations of a toddler
12-15 month: IPV, PCV, MMR, varicells, HIB
12-23 months: Hep A 2 doses 6 months apart
15-18months: dtap and annual flu

Nutrition of a toddler
breast milk or formula through 1 year, 1-2 year whole milk, after 2 can transition to low fat, limit juice consumption to 4-6oz per day, prevent choking, nuts, grapes, hot dogs, peanut butter, raw carrots, tough meat, popcorn

What are safety hazards of toddlers?
burns, drowning, falls, aspiration, prevention

A nurse is assessing a 2.5-year-old toddler at a well-child visit. Which of the following findings should the nurse report to the provider?
A. height increased by 7.5cm or 3inches in the past year
B. Head circumference exceeds chest circumference
C. anterior and posterior fontanels are closed
D. current weight equals four times the birth weight
B

A nurse is performing a develomental screening on an 18 month old. Which of the following skills should the toddler be able to perform? (select all that apply)
A. build a tower with 6 blocks
B. Throw a ball overhead
C. walk up and down stais
D. draw circles
E. use a spoon without rotation
B, E

A nurse is providing teaching about age-appropriate activities to the parent of a 2 year old. Which of the following statemetns by the parent indicates an understanding of the teaching?
A. I will send my child’s favorite studdef animal when she will be napping away from home
B. My child should be able tot stand on one foot for a second
C. The soccer team my child will be playing on starts next week
D. I should expect my child to be able to draw circles
A

A nurse is providing anticipatory guidance to the parents of a toddler. Which of the following should the nurse include? (select all that apply)
A. Develop food habits that will prevent dental caries
B. Metting caloric needs resulting in an increased appetite
C. expression of bedtime fears is common
D. Expect behaviors associated with negativism and ritualism
E. Annual screenings for phenylketonuria are important
A, C, D

Growth of preeschoolers
4.5-6.5 pounds per year
2.3-3.5 inches per year or 6-9cm

Gross motor skills of 3 year old preeschooler
can ride tricycle and jump off bottom step on stairs

Gross motor skills of 4 year old preschooler
can skip and hop on one foot and throw the ball over head

Gross motor skills of 5 year old preschooler
can jump rope

Cognitive development of the preschooler?
Piaget, preoperational phase 4-7 years, moving from preconceptual phase to the phase of intuitive thought, magical thinking, animisim, centration, time

Erikson, initiative vs guilt

Regression
in preschoolers, another baby in the family can cause the preschooler to regress to bed wetting or thumb sucking, to be expected

What are appropirate activities for a preschooler?
playing ball, puzzles, tricyles, dress up, role playing

Immunizations of preschooler
4-6 years, dtap, mmr, IPV and annual flu

What does the sleep schedule of a preschooler look like?
12 hours of sleep, bedtime routine

Teeth of preschooler
eruption of primary teeth is finalized by the beginning of the preshool years

What may be a safety initiative for preschooler?
protective gear with tricycles

A nurse is providing teaching to the parent of a preschool age shcil about methods to promote sleep. Which of the following statements by the parent indicated an understanding of the teaching?
A. I wil sleep in the bed with my child if she wakes up during the night
B. I will let my child stay up and additional 2 hours on weekend nights
C. I will et my child watch television for 30 minutes nust before bedtime each night
D. I will keep a dim lamp on in my child’s room during the night
D

A nurse is conducting a well child visit with a 5 year old child. Which immunizations shoudl the nurse plan to administer to the child? (Select all that apply)
A. DTaP
B. IPV
C. MMR
D. PCV
E. Hib
A, B, C

A nurse is preparing an education program for a group of parents of preschool-age children about promoting optimum nutrition. Which of the following information should the nurse include in the teaching?
A. saturated fats should equal 20% of total daily caloric intake
B. Average calorie intake should be 1800 calories per day
C. dailyintake of fruits and vegetables should total 2 servings
D. Healthy diets include a total of 8g of protein each day
B

A nurse is performing a developmental screening on a 3 year old child. WHich of the following skills should the nurse expect the child to perform?
A. ride a tricycle
B. Hop on one foot
C. jump rope
D. throw a ball overhead
A

A nurse is caring for a preschool age child who says she needs to leave the hospital because her doll is scared to be at home alone. Which of the following characteristics of preoperational thought is the child exhibiting?
A. Egocentrism
B. Centration
C. Animism
D. Magical thinking
C

Growth of school age children
4-6 pound weight gain per year and grown 2 inches (5cm) per year, permanent teeth start to come in

Piaget cognitive development of school age children
concrete operations, perceptual to conceptual thinking, learsn to tell time, see other perspectives, solve problems

Erikson’s stage of school age children
industry vs inferiority, trying to make meaning contributions to society and cooperative and compete with others, peer groups important, competitive and cooperative play

What types of activites are appropriate for school age children?
board games, hop scotch, bikes, jump rope, organized sports

Immunizations of school age children
11-12 years: DTaP, HPV vaccine (series of 3 shots)

How much sleep is recommended for a 12 year old?
9 hours of sleep

What is a safety measure for school age children?
helmets

A nurse is discussing prepubesence and preadolescen with a group of parents of school-age children. Which of the following information should the nurse include in the discussion?
A. initial phsyciologic changes appear during early childhood
B. changes in heigh and weight occur slowly during this period
C. growth differences between boys and girls become evident
D. signs of sexual maturation become highly visible in boys
C

A nurse is conducting a well child visit with a child who is scheduled to recive the recommended immunizations for 11 to 12 year olds. Which of the following immunizations should the nurse administer? (select all that apply)
A. TIV
B. PCV
C. MCV4
D. Tdap
E. RV
A,C, D

A nurse is teaching a course about safety during the school age years to a group of parents. Which of the following information should the nurse include in the course? (select all that apply)
A. gating stairs at the top and bottom
B. wearing helmets when riding bicyles or skateboarding
C. riding safely in bed of pickup trucks
D. implementing firearm safety
E. wearing seat belts
B, D, E

Growth in adolescents
girls stop growing 2-2.5 years after their period starts, boys stop growing abotu 18-20

Sexual maturation is adolescent girls
breast development, pubic hair growth, underarm hair, period

Sexual maturation in adolescent boys
testicular enlargement, pubic hair, penil enlargement, underarm hair growth, facial hair, vocal changes

Piaget cognitive stage of development for adolescents
formal operations

Erikson’s stage of development for adolescents
identity vs role confusion

What are appropriate activites for adolescents
video games, music, sports, pets, reading

Immunizations for adolescents
flu, 16-18 years, meningitis before college

Injury prevention for adolescents
helmet use, seat belts, driving, susbtance abuse

Safe medication administration for children
oral is preferred, smallest measuring device possible, dont mix oral meds in formula, put in side of mouth, hold cheeks, and stroke chin to swallow

Administration of ear drops
pinna down and back

IM injections
preffered route is vastus lateralis, then the ventral gluteal or in the deltoid, 22-25 guage with half inch to 1 inch needle

IV safe administration
procedure room, away from bed, EMLA cream to numb area is recommneded, avoid terms like bee stink or stick, keep stuff out of sit, parents can stay, swaddle the infant, non-nutritive sucking is offered before, during, and after to infants

A nurse is providing teaching about expected changes during puberty to a gorup of parents of early adolescent girls. Which of the following statements by one of the parents indicates and understanding of the teaching?
A. girls usually stop growning abotu 2 years after menarche
B. girsl are expected to gain about 65 pounds during puberty
C. girls experience menstartion prior to breast development
D. Girls typiucaly grow more than 10 inches during puberty
A

A nurse is providing anticipatory guidance to the parent of a 13 year odl adolescent. Which of the following screenings shoudl the nurse reccomend for the adolescent? (select all that apply)
A. body mass index
B. blood lead level
C. 24 hour dietary recall
D. Weight
E. Scoliosis
A, D, E

A nurse is caring for an adolescent whose mother expresses ocncersn about her child sleeping such long hours. Which of the following conditions shoudl the nurse inform the mother as requiring additional sleep during adolescnets?
A. sleep terrors
B. rapid growth
C. elevated zinc levels
D. slowed metabolism
B

A nurse is teaching class about puberty in boys. Which of the following should the nurse include as the first manifestation of sexual maturation?
A. pubic hair growth
B. voval changes
C. testicular enlargement
D. facial hair growth
C

A nurse is planning to administer the influenza vaccine to a toddler. Which of the following actions should the nurse take?
A. administer subq to the abdomen
B. use a 20guage needle
C. Divide the medicaiton into two injections
D. place the child in supine position
D

A nurse is preparing to administer an IM injection to a child. Which of the following muscle gorups is contraindicated?
A. Deltoid
B. ventrogluteal
C. vastus lateralus
D. dorsogluteal
D

A nurse is teaching a parent of an infant about administration of oral medications. Which of the following should the nurse include in the teaching? (select all that apply)
A. use a universal dropper for medication administration
B. as the pharamacy to add flavoringto the medication
C. add the medication to a formula bottle before feeding
D. use the nipple of a bottle to administer the medication
E. hold the infant in a semireclining position
B, D, E

A nurse is preparing to administer medication to a toddler. Which of the following actions should the nurse take? (select all that apply)
A. identiy the toddler by asking the parent
B. tell the parent to administer the medication
C. calculate the safe dosage
D. ask the toddler what toy he wants to hold during administration
E. offer juice after the medication
C, D, E

A nurse is caring for an infant who needs otic medicatioin. Which of the following is an appropirate action for the nurse to take?
A. Hold the infant in an upright position
B. pull the pinna downward and straight back
C. hyperextend the infants neck
D. ensyure that the medication is cool
B

Pediatric pain management
self report is only used for children 4 and older, FLACC scale is 2m to 7 years, pain rate ona scale of 0-10 assessing behaviors of the child
FACES: 3 years and odler
Oucher scale: 3-13
Numeric scale: 5 and older
use play therapy to epxlain procedures
ive medications to kids routinely versus prn
combining opioid and non-opioid medications
EMLA cream, apply 1 hour prior to small stick or 2.5 hours before a big stick, occlusive dressing over it

A nurse is competing a pian assessment of an infant. Which of the following pian scales should the nurse use?
A. FACES
B. FLACC
C. Oucher
D. Non-communicating childrens pain checklist
B

A nurse is planning care for achild following a surgical procedure. Which of the following interventions dhoul the nurse incldue in the plan of care?
A. administer NSAIDS for a pain grater than 7 on a scale fo 0 to 10
B. administer intranasal analgesics PRN
C. Administer IM analgesics for pain
D. administer IV analgesics on a schedule
D

A nurse is assessing an infant. Which of the following are manifestations of pain in an infant? (select all that apply)
A. Pursed lips
B. loud cry
C. lowered eybrows
D. Rigid body
E. pushes away stimulus
B,C,D

A nurse is planning care for an infant who is experincing pain. Which of the following interventions should the nurse include in the plan of care? (select all that apply)
A. offer a pacifier
B. use of guided imagery
C. use swaddling
D. initiate a behavioral contract
E. encourage kangaroo care
A,C,E

A nurse is preparing a toddler for an IV catheter insertion using atraumatic care. Which of the following actions shoudl the nurse take? (select all that apply)
A. explain the procedure using the child’s favorite toy
B. ask the paretns to leave during the procedure
C. perform the procedure with the child in his bed
D. allow the child to make on choice regarding the procedure
E. apply lidocaine and prilocaine cream to three potential insetion sites
A, D, E

Hospitalization, illness, and play for the infant
stranger anxiety 6 to 18 months

Hospitalization, illness, and play for the toddler
behavior may regress, separtion anxiety, intense reaction to procedure, parallel play

Hospitalization, illness, and play for the preschooler
magical thinking, they may think they caused an illness to happen, still experience separtion anxiety, explain the procedure in very simple clear language, give them a choise if possible (cup or spoon), associate play, paly together without much coordination

Hospitalization, illness, and play for the school age child
describe pain and increased ability to understand cause and affect, give factual info, tell the truth, encouage contact with peer groups, and express feelings, cooperative play, play in groups, more organized

Hospitalization, illness, and play in the adolescent
body image disturbance, feels isolated from peers, give factural info, tell the truth, encoruage contact with peer group, and express feelings, friends can come visit

A nurse is caring for a preschooler. Which of the following is the expected behavior of a preschool-age child?
A. Describing manifestations of illness
B. relating fears to magical thinking
C. understanding cause of illness
D. awareness of body functioning
B

A nurse on a pediatric unit is caring for a toddler. Which of the following behaviors is an effect of hospitalization? (select all that apply)
A. believes the experience is a punishment
B. experiences separtion anxiety
C. displays intense emotions
D. axhibits regressive behaviors
E. Manifests disturbance in body image
B, C, D

A nurse is teaching a parent about parallel play in children. Which of the following should the nurse include in the teaching?
A. children sit and observe others playing
B. Children exhibit organized play when in a group
C. the child plays alone
D. The child plays independently when in a group
D

A nurse is teaching a group of parents about separation anxiety. Which of the following information shoudl the nurse include in the teaching?
A. it is often observed int the school age child
B. detachment is the stage exhibited in the hospital
C. it results in prolonged issues of adaptibility
D. kicking a stranger is an example
D

anticipatory grief
when death is expected or a possible outcome

complicated grief
extends for more than 1 year following the loss

parenteral grief
intense, long lasting, and complex

sibling grief
differs from adult/parenteral; depend on age and developmental stage

Infant/toddler view of death/dying
no concpet of death

Preschoolers(3-6) view of death/dying
magical thinking, may feel guilt or shame, views dying as temporary

School age (6 to 12) view of death/dying
adult concept of death, express fear through uncooperative behavior

Adolescent (12-20) view of death and dying
adult concept of death, resul of peers vs parents, stressed out by changes in physical appearance

Physical manifestations of death
senstaion of heat when body feels cool, decreased sentation, loss of sesnes, decrease LOC, swallowing issues, bradycardia, hypotension, Cheyne stokes respirations

Nursing responsibilty after death
allow family to stay with body, rock infant/todderl, assist in post-mortem care

A nurse is caring for child who is dying. Which of the following are findings of impending death? ( select all that apply)
A. heightened sense of hearing
B. tachycardia
C. difficulty swallowing
D. sensation of being cold
E. cheyne-stokes respirations
C, E

A nurse is teaching a parent about complicated grief. Which of the following statements should the nurse make?
A. it is considred complicated gried if you are still grieving after 6 months
B. personal acitciites are affected wehn expericining complicated grief
C. parents will expeirce complicated grief together
D. complicated grief self-resolved in 12 months
B

A nurse is teaching a parent of a preeschol child about factors that affect the child’s perception of death. Which of teh following factors should the nurse include in the teaching?
A. Preschool children have no concept of death
B. Preschool children percieve death as temporary
C. preschool children often regress to an earlier stage of behavior
D. preschool children experience fear related to the disease process
B

A nurse often care for children who are dying. WHich of the following are approporate actions of rhte nurse to take to maintian professional effectivness? (select all that apply)
A. remain in contact with the family after thier loss
B. develop a professional support system
C. take time off from work
D. suggest that a hospital representative attend the funeral
E. demonstrate feelings of sympathy toward the family
B, C

A nurse is caring for a child who has a terminal illnes and revies palliative care with an assistive personel (AP). Which of the following statements by the AP indicates understanding of this review?
A. im sure the family is hopeful that the new medication will stop the illness
B. Ill miss wokring wit this client now that only nurses will be caring for him
C. I will get all the clients personal object out of his room
D. I will listen and respons as the family talks about thier child’s life
D

Meningitis
Viral often resolves with supportive care, bacterial is more dnagerous, PCV and Hib vaccine help prevent, s/s include photophobia, n/v, irritability, h/a
newborns: poor muscle tone, weak cry, refusal to eat, vomiting, diarrhea, poor sucking, possible fever or hypothermia, neck is supple without nuchal rigidity, buldging fontanels are a late sign

3m to 2years: siezures with a high pitch cry, fever and irritability, bulding fontanels, nuchal rigicity, poor feeding adn vommting

2 years to adolescents: siezures, nuchal rigicity, fever and chilld, headache, n/v, irritabiltiy, petechia, positive brudsinksi’s sign (puill head forward – extremities will also flex (bro why are you pulling on my neck)
positive kernigs signs – leg is flexed and you try to extend, it causes pain

Meningitis labs
CSF analysis through lumbar puncture, empty bladder before, EMLA cream, side lying position, after procedure remain in bed 4-8 hours in a supine position

Bacterial: cloudy, increased WBC, increased protein, decrease glucode and +gram stain

Viral: clear, slightly elevated WBC, noraml or slightly elevated protein, normal glucose, negative gram stain

Nursing managment for meningitis
it it is suspected put them into droplet precautions, decrease LOC = NPO, provide quite environment, dim the lights, siezure preautions, bacterial will need IV abx, maintian contact precautions for bacerial for 24 hours after abx treatment has started, monitori for increased ICP**
infants: bulging fintanels, increase in head circumference, high pitch cyr, bradycarida, adn respiraty changes
childre: irritability, headahce, n/v, siexures, braduycardia, adn respiratoruy changes

Reyes syndrome
liver dysfunction and cerebral edema, associated with giving hcildren aspirin for a fever, follows a viral illness like the flu, gasterentritis, or varicella, lab tests include: liver enzymes (AST/ALT increase), increased ammonia levels, liver biopsy for diagnosis, CSF analysis
s/s: lethargy, irritiabilty, confusion, deliriu, vomtimg, LOC

A nurse is caring for a client who has suspected meningitis and a decreased level of consiousness. Which fo the following actions by the nurse is appropriate?
A. place the patient on NPO status
B. prepare the client for a liver biopsy
C. position the patient in dorsal recumbent
D. put the client in a protective environment
A

A nurse is reviewing cerebrospinal fluid analysis for a client who has suspected meningitis. Which of the following findings shoudl the nurse identify as indicating viral mengingits? (select all that apply)
A. negative gram stain)
B. normal glucose content
C. Cloudy color
D. decreased WBC count
E. normal protein count
A, B, E

A nurse is caring fro a 4 month old infant who has meningitis, Which fo the following findings is associated with this diagnosis?
A. fepressed anterior fontanel
B. constipation
C. presence of rooting reflex
D. high pitched crying
D

A nurse is caring for a patient who possibly has Reye syndrome. Which of the following is a risk factor for developing reye syndrome?
A. recent hisotyr of infectious cystitis cauased by candida
B. recent hisotrial of bacterial otitis media
C. recent epidose of gastrenteritis
D. Recent episode of Haemophilus influenzae meningitis
C

A nurse is developing an inservice about viral and bacterial meningiting. The nurse shoudl include that the introduction of which of the following immunizations decreased the incidence of bacterial meningitis in children? (select all that apply)
A. IPV
B. PCV
C. Dtap
D. Hib
E TIB
B, D

Risk factors for siezures
cerebral edema, fever, trauma, bleeding, toxins in body (lead), hypoglycemia, electrolyte imbalance, infection

What are the three phases of tonic-clonic siezures
tonic- arms and legs flex up and head and neck extend; stiff, LOC
clonic- jerking movements
postical- awake and confused

absense seizure
school age childre (4-12), loss of consiousness 5-10 seconds, daydreaming look, drop whats in hands, lip smacking or twitching of face

myoclonic
no postical

seizure diagnosis
EEG to find cause, prior to EEG no caffien and wash hair

seizure medications
carbamasepine, valporic acid, phenytoin, and diazepam

Complications of siezures
status epilepticus which a a seizure lasting longer than 30 minutes, medical emergency

A nurse is caring for a child who has absence seizures. Which of the following findings should the nurse expect? (select all that apply)
A. loss of consciousness
B. appearance of daydreamimg
C. dropping held objects
D. falling to the floor
E having a piercing cry
A, B, C

A nurse is caring for a child who just experienced a generalized seizure. Which of teh following is the priority actio for the nurse to take?
A. Maintain the child in a side ying position
B. loosen the childs restricitve clothing
C. reorient the hcild to the environment
D. not the time and characteristics of the seizure
A

A nurse is providing teaching to the parent of a child who is to have an EEG. Which of the following responses hodul the nurse include in the teaching?
A. decaffieicated beverages should be offred on the morning of the procedure
B. do nto wash your child’s care the night before the procedure
C. withold all foods the morning of the procedure
D. give your child an analgesic the night before the procedure
A

A nurse ic teaching a group of parents about the risk factors for seizures. Which of the following factors shoudl the nurse include in the teaching? (select all that apply)
A. febrole episodes
B. hypoglycemia
C. sodium imbalances
D. low serum lead levels
E. presence of diphtheria
A, B, C

A nurse is reviewing treatment options with the parent of a child who has worsening seizures. Which of the following treatment options shudlt hte nurse include in the discussion? (select all that apply)
A. vagal nerv stimulator
B. additional antiepeltic medications
C. corpus callosotomy
D. focal resection
E. radiation therapy
A, B, C, D

minor head injury
confusion, vomiting, pallor, irritability or drowsiness, irritability is sually one of teh first signs of increased ICP

infant signs of head injury
buldging fontanels, high pitched cry, poor feeding, increased sleeping, reslessness, setting sun sign, distended scalp veins

children signs of head injury
nausea, vomtiing, headache, seizures, blurred vision

late signs of head injury
delayed or impaired pupillary responses, posturing, decreased response to painful stimuli, cheyne-stokes respirations, optic dis swelling, decreased in LOC

interventions for head injury
stabalize the spine first, then vital signs, GCS, keep HOB 30, maintin head in midline/neutral position, minimize oral or endotracheal suctioning, avoid coughing or blowing their nose, foley catheter, stool softeners, implemet seizre precautions

medications for head injuries
corticosteroids, mannitol (diuretic), anti-epileptics

surgical interventio for head injury
craniotomy, complications include hemorrhage, brain herniation, s/s of hernia include loss of blinking, loss of gag reflex, unreactive pupils, coma, or resp arrect

A nurse is in the emergency department assessing a child following a motor vehicle crash. The child is unresponsive, has spontaneous respirations of 22/min, adn has a laceration of the forehead that is bleeding. Whic of the following nursing actions should be first?
A. Stabalize the neck first
B. cleanse the child’s laceration with soap and water
C. implement siezure precautions for the child
D. initiate Iv access for the child
A

A nurse is caring for an adolescent who has a closed head injury. Which of the following findings are indications of increased ICP? (select all that apply)
A. report of headache
B. alteration in pupillary response
C. increase motor response
D. increased sleeping
E. increased senory response
A, B, D

A nurse is caring for a child with ICP. Which of the following actions shudl the nurse take? (select all that apply)
A. suction the endotracheal tube every 2hours
B. maintian a quiet environment
C. use two pillows to elevate the head
D. administer a stool softener
E. maintian body alignment
B, D, E

A nurse is assessing a child who has a concussion. Which of the following findings should the nurse expect. (select all that apply)
A. amnesia
B. systemic hypertension
C. bradycardia
D. respiratory depression
E. confusion
A, C, D, E

A nurse is caring for a child who is taking mannitol for cerebral edema. Which of the following adverse effects shoudl the nurse monitor the child for and report to the provider?
A. bradycardia
B. weight loss
C. confusion
E. constipation
C

snelling test
stand 10 feet away

myopia
near sightedness

hyperopia
far sightedness

strabismus
inward or outward deviation of one of the eyes, treatment is to patch the good eye

acute otitis media
middle ear infection, common under 7, their tubes are shorter and more horizontal than adults, give pain meds, abx, and if surgery is needed, tube is put in the middle ear, myringotomy, and the placement of tympanoplasty tubes, tubes will fall out on their out in 2-6 months and tel HCP when they come out, dont get thier ears wet

Desired SpO2
95-100%

Metered dose inhalers
shake 5-6 times, spacers make them more effective, hold inhaler up, whil pressing in, take a slow deep breath, and hold breath for 10 seconds before exhaling.

dry powder inhaler
don’t shake

Chest physiotherapy
schedule 1 hour before or 2 hours after a meal to prevent vomiting, give bronchiodilator or nep treatment prior

Hypoxemia
s/s include tachypnea, tachycardia, restlessness, accessory msucles, nsal flaring

O2 toxicity
leads to hypoventilation and maybe LOC

suctioning
clean technique fo nasal and oral

ET tube and trach tube suctioning
high fowlers or fowlers, catheter one half of the diamete, hyperoxygenation and hyperventilate with 100%, surgical aspetic, limit suctioning to 5 seconds for infants and 10 seconds for children, rest for 30to60 seconds in between passes

tonsilittis
fever meds, abx to cure the infection, culture to test for strep, tonsilectomy, side lying position initially, assess for bleedig, frequent swalloing and clearing of the throat, clear fluids afer hag reflex has returned, no citrus juices, no milk products, discourage coughing or nose blowing, warn parten that there may besome blood clots, limit strenious acitivty, full recovery in about 2 weeks, grow beta of strep ca laeas to kidney infection or rheumatic fever

bacterial epiglottitis
drooling, hoarseness, difficulty speaking and swallowing, and high fever, most important, do not put anything in their throat, no throat culture, or tongue balses, cause airway to lcose up, abx therapy, intubation supplies ready

influenza
fever, body aches, congestion, antiviral (usually within first 48 hours)

complications of acute and infectious respiratory illnesses
pneumothorax and pleural effusion

bronchodilators for asthma
albuterol -> s/e is tachycardia and temors

anticholinergic like ipotropirum for asthma
s/e cant see, cant see, cant pee, cant spit, and shit

steroid for asthma
prednisode, rinse mouth after steroid inhaler because they can get a fungla infection

peak flow meters
stand up, 0 out machine, lips around device, blow out hard, 3x, highest reading

complications of asthma
status asthamticus, not relived by medications, intubation

cystic fibrosis
caused by a genetic mutation, autosomal recessive, both parent shave to carry recessvie traits, iincreased thick tenacious mucous, pancrea, lings, liver, small intestines, and reporductive organs
-carrel chest, finger clubbing, large loose fatty fould smelling stools (steatorrhea), not gain weight, delayed growth, failure to thrive, deficieny of fat-soluble vitamines (ADEK), sweat and tears are salty

diagnosis of cycstic fibrosis
sweat chloride test and DNA testing

cystic fibrosis treatment
Iv abx, and o2 therapy, diet high in calories, and protein, pancreatic enzymes with thier meals to help with digestion, pancrelipase and vitamin supplements, albuterol, anticholinergics, dornase afa (decreases the viscocity of the mucous), chronic managing, prents find support groups

Congenital heart defects usually result in 2 things
hypoxemia and heart failure, s/s include tachypnea, dyspnea, tachycardia, peripheral edema, cyanosis, exercise intolerance, and polycythemia (increase in rbcs)

Increasing pulmonary blood flow defects

  1. ventricular septal defect (VSD) a. creates a harsh murmur that can be heard at the left sternal border 2. atrial septal defect (ASD) a. loud hard murmur; split sound 3. patent ductus arteriosus (PDA) a. creates bounding pul

Obstructive blood flow defects

  1. pulmonary stenosis a. systolic ejection murmur 2. aortic stenosis 3. coarctication of the aorta * a. upper body: bounding pulses and high pressure, flushed warm skin b. lower body: low pressure, faint pulses, cool skin

decreasing pulmonary blood flow defects

  1. tricuspid atresia a. complete closure of the tricuspid valve also have to have an ASD 2. tet of fallot * PROV a. pulmonary stenosis, VSD, overriding aorta, and right ventricular hypertrophy

Mixed blood flow defects

  1. transposition of the great arteries a. sx within first 2 weeks of life – major cyanosis 2. truncus arteriosus a. no spetum between the ventricles b. requires sx after birth 3. hypoplastic left heart syndrome

EKG, echo, cardiac catheterization
-Allergies to shellfish or iodine
-NPO 4-6 hours prior to procedure
-Both pedal pulses located
-Assess insertion site for bleeding
-Flat position 4-8 hours post-op

Nursing care of cardiovascular disorders
o Frequent rest periods; cluster care
o Small frequent meals
o Crying kept to a minimum
o Encourage semi-fowlers or fowlers
o Car seat at 45-degree angle vs flat
o Feed Q3 hours
o Enlarged opening on bottle nipple

Medications for cardiovascular disorders
o Digoxin – help improve contractility of the heart Toxicity – n/v, halo. Decreased HR and appetite o ACE (Aprils) – help provide vasodilation o Beta blockers – helps decrease HR, BP, and causes vasodilation o Lasix – K levels High potassium foods *** look in nutrition Severe hypoxemia episode knee to chest to calm them down High risk for bacterial endocarditis – abx prior to dental and surgical procedures

Rheumatic fever
Inflammation of the heart, blood vessels, and joints Caused by a strep throat infection untreated or partially treated o 2-6 weeks following Lab tests – throat culture, serum ASO titer, EKG, diagnosis based on the jones criteria (pt. needs to have 2 major criteria or 1 major and 2 minor) o Major criteria: carditis, subcutaneous nodules (non-tender), polyarthritis, rash (pink-non pruritic on the trunk and the inner surfaces of the extremities), Chorea – involuntary muscle movements o Minor: fever + pain in one joint

Kawasaki disease
Acute systemic vasculitis – inflammation of the blood vessels Acute phase o Onset of high fever that is unresponsive to meds, with development of other cm’s o Irritability, red eyes without drainage, bright red chapped lips, strawberry tongue, red oral mucosa, red palms and feet, joint pain, enlarged lymph nodes, etc, Subacute phase o Resolution of fever and gradual subsiding of other cm’s o Peeling skin Convalescent phase o No cm’s seen except abnormal labs o Resolution 6-8 weeks from onset Treatment – IV Igg ** (gamma globulin) o + aspirin o Avoid live immunizations for 11 months after the onset of the disease

Epistaxis
nose bleeding Sit upright and lean forward, pinch the nose for 10 min until bleeding stops, ice on the nose, or cotton or tissue in the nare After the bleeding stop – Vaseline in the nose to help prevent re-bleeding + recommend parent uses cool mist humidifier

Iron deficiency anemia
Poor diet, drink a lot of cow milk (low in iron) Lab work – RBC, hbg & hct low Diet in iron, protein, and vitamin C Iron supplement 1 hour before or 2 hours after milk or antacids, vitamin C, straw, IM injection use z track method, stool is expected to turn tarry green color if dose is adequate, brush teeth after Prevent overdosing of iron – locked in cabinet

Sickle cell anemia
Autosomal recessive genetic disorder HbS is produced Increased blood viscosity, obstruction of blood flow, tissue hypoxia Painful af Crisis – exacerbation African American highest risks Fam history, reports of pain, SOB, pallor, jaundice (destruction of RBC’s) Vasoclusive crisis 4-6 days painful ischemia in the tissue Treatment: fluids *** and pain control o Blood products Complications CVA and any reactions to the blood products

Hemophilia
Prolonged bleeding time due to lack of a specific factor that’s needed to clot properly A = lack of factor 8 B = lack of factor 9 Excessive bleeding, joint pain and stiffness, bruising Labs: prolonged PTT, platelets and prothrombin will be normal No rectal temps, avoid skin punctures when necessary, hold pressure 5 min, painful joints = elevate and apply ice to that area Replace the factors that are missing Minimize the risk of bleeding o RICE Complications: joint deformity

Rotavirus

  • most common cause of diarrhea in kids <5 o Vaccine available o s/s: watery diarrhea, vomiting, and fever

Pin worm (Enterobius vermicularis)

  • cause perianal itching o Tape test for diagnosis while sleeping

When a kid has diarrhea, you need to know what foods/drinks they should and shouldn’t have *
o Should have oral rehydration therapy drink (ORT) o Shouldn’t give fruit juices, carbonated drinks, jello, caffeine, chicken or beef broth & no BRAT diet

Dehydration
o Mild – slight thirst and capillary refill is a little longer o Moderate – cap refill between 2-4 seconds, thirst and irritability, dry mucous membranes, tears and skin turgor are decreased o Severe – cap refill >4 seconds, tachycardia, extreme thirst, mm very dry, tented skin, no tearing, sunken eyes, sunken anterior fontanel, oliguria or anuria

Cleft lip
repaired in 2-3m o Before sx: use a wide based nipple for feeding, encourage breastfeeding, squeeze cheeks together to decrease gap o After sx: back and upright, elbow restraints, ns water or diluted hydrogen peroxide to clean the suture line, antibiotic ointment if prescribed

Cleft palate
repaired in 6-1m o Before sx: upright for feeing, one-way valve bottle with specially cut nipple for feeding, burp frequently o After sx: prone position, IV fluids then clear liquid for first 24 hours, nothing in mouth that could mess up the sutures, elbow restraints

Complications of cleft
ear infections and hearing loss – seen by specialists, Speech therapists o Dental problems – teeth may not erupt normally

GERD – gastrointestinal reflex disease
Usually self resolves by 1 year of age s/s with infants – spitting up, irritability, excessive crying, blood in the vomit, arching of back, stiffening, resp. problems, FTT, apnea s/s with children – heartburn, abd pain, difficulty swallowing, chronic cough, noncardiac chest pain nursing care small frequent meals, thicken infant’s formula with rice cereal, avoid foods that can make it worse, head elevated for 30 at least 1 hour after eating o meds: PPI (omeprazole, pantoprazole); H2-receptor antagonists (famotidine or ranitidine) Sx – nissen fundoplication

Pyloric stenosis
thickening of the pyloric sphincter = obstruction Projectile vomiting, dehydration and constant hunger, olive shaped mass in the RUQ Sx – pylorotomy

Hirschsprung’s Disease
congenital aganglionic megacolon structural anomaly of the GI tract caused by lack of ganglion cells in the segments of the colon resulting in decreased motility and mechanical obstruction Ribbon like stool, vomiting bile, abd distention Risk factor infant fails to pass meconium in 24-48 hours of life Nursing interventions – high calorie, protein & low fiber diet o Surgery remove bad portion of the colon & may need a colostomy for a while

Intussusception

  • proximal segment of the bowel telescopes into a more distal segment, resulting in lymphatic and venous obstruction causing edema in the area with progression, ischemia and increase mucous into the intestine will occur Common in infants and children 3m-6 years Red currant jelly stool, sausage shaped abd mass CF at risk Air enema is therapeutic procedure

Appendicitis
Avg age is around 10 years Abd pain in the RLQ, ↓ or absent bowel sounds, fever, WBC ↑and inflammatory markers Diagnosis with CT Avoid heat to the abd Fluids, abx prior to the stomach Suddenly feeling better – ruptured appendix medical emergency

Enuresis
uncontrolled or unintentional urination after the age of 5 years for at least 3m’s Primary – never had control of bladder Secondary – have been potty trained and now they are wetting their bed o Regression from stress of emotional trauma Self-esteem and coping strategies Restrict fluids in the evenings, avoid constipation, etc.

UTI’s
Frequent urination, foul smelling urine, fever, pallor, poor appetite, vomiting, increase in thirst, swelling of the face, and seizures Diagnosis – urinalysis nitrates and leukocytes elevation Educations – females wipe front to back, cotton underwear, avoid bubble baths, void more, empty bladder fully Important constipation – high fiber diet

Bladder exstrophy
the bladder/urethra/ureteral orifices are coming through the suprapubic area – medical emergency; requires immediate surgery Sterile gauze over that area and prepare for sx

Hypospadias
Urethral opening is on the underside (ventral) side of the penis No circumcision

Epispadias
Urethral opening on the upper part of the penis (dorsal) side No circumcision

Phimosis
narrowing of the opening of the foreskin can’t retract the foreskin

Cryptorchidism
undescended testes Sx at 6 and 24m’s

Hydrocele
fluid in the scrotal sac

Testicular torsion

  • medical emergency Enlargement of the effected testical and severe and sudden onset of pain

Acute glomerulonephritis [AGN]
Associated with a strep infection Cloudy tea colored urine, dec urine output, periorbital edema, facial edema that’s worse in the am and then spreads down over the day, mild -severe HTN, oliguria Proteinuria and Smokey or tea colored urine, hematuria, increased specific gravity, ASO titer – for strep infection Nursing care o Restrict sodium and fluid o Edema risk for skin breakdown o Diuretics, anti-hypertensives, abx for strep infection

Nephrotic syndrome
alteration in the glomerular membrane that allows proteins [esp. albumin] to pass through to the urine resulting in decreased serum osmotic pressure Facial and periorbital edema, dec urine, frothy urine, norm BP, >2+ protein, hypoalbuminemia, hyperlipidemia, hemoconcentrion, hyponatremia maybe Daily weights – same scale, same time, same amount of clothing on everyday Monitor edema – measure abd girth @ the level of the umbilicus Restrict fluids and salt Skin breakdown Meds – steroids monitor for GI bleeding, hyperglycemia, etc o Albumin and diuretics help increase the plasma volume and decrease edema in the pt.

Fractures
Open or compound – bone is sticking out of the skin Closed or simple – bone not sticking out Complicated – organ or tissue is also damaged ABC’s + elevated the extremity, apply ice, stabilize the injured area & a complete neurovascular check o Sensation o Skin temp. o Skin color o Cap refill o Pulses o Movement Casting – elevate the cast above the level of the heart for the first 24-48 hours, apply ice for 24 hours to dec swelling & turn and position the pt. every 2 hours to help dry the cast, assess for inc warmth or hot spots on the cast – hot spot indicated infection Plaster casts use the palms of your hands to avoid denting Expose all surfaces to promote drying Don’t put anything in the cast to itch

Traction care
Align, mobilize, and reduce muscle spasms in patients who have fractures Maintain body alignment, give meds to help prevent muscle spasms and pain, neurovascular checks, pin sites for s/s of infection, make sure the weights hang freely & not on the bed or floor, do not lift or remove weights unless ordered Halo tractions – wrench attached to the vest if needed for CPR

Compartment syndrome
compression of the nerves, blood vessels, and muscle within a confined space Tissue necrosis can occur Very intense pain unrelieved with meds, numbness, pulselessness, inability to move digits, pallor, cool extremities Fasciotomy muscle compartment is cut open to allow tissue to swell, decrease pressure, and restore blood flow 5 P’s – pain, paresthesia, pulselessness, paralysis, and paleness

Osteomyelitis
open/compound fractures – infection of the bone Fever, pain, tachycardia, edema Bone biopsy Abx therapy to treat

Clubfoot
Treatment is serial casting

Legg-calve-perthes disease
aseptic necrosis of the femoral head (uni or bi) Intermittent painless limp, hip stiffness, shortening of the effected leg, limited ROM Bracing, casting, or traction or replacement of the hip joint

Developmental dysplasia of the hip [DDH] *
Infants – asymmetry of the gluteal and thigh folds, limited hip abduction o + ortolani test hip is reduced by abduction o + barlow test hip is dislocated by adduction Children – one leg is shorted than the other, walk with a limp, + tendelenberg sign [while bearing weight on the effected side, the pelvis tilts down], walks on tippy toes on one foot Newborn to 6 months = Pavlik harness o 12 weeks o Check straps every 1-2 weeks for adjustments by HCP o Preform neurovascular and skin checks o Use an undershirt and wear knee socks o Gently massage under the straps o No lotion or powders o Put diaper on under the straps Over 6 months o Bryant harness Hips flexed at a 90-degree angle with the butt raised off the bed Maintain traction & assure alignment Skin care o Hip spica cast Neurovascular checks Position casts on the pillow & keep elevated until dry Frequent position changes to promote even drying Handle casts with palm of hands to prevents dents until dry Give sponge baths to avoid wetting the cast Use waterproof barrier around the genital opening so nothing gets in there Complications from casts and harnesses – bowel and bladder eliminations Fiber to help pooping and fluids

Osteogenesis imperfecta
an inherited condition that results in bone fractures and deformity along with restricted growth Heterogeneous autosom dominant Brittle bone disease Multiple bone fracture, blue sclera, early hearing loss, small discolored teeth No cure – treatment is supportive Medication pamidronate: can be used to increase bone density o s/e: hypocalcemia, hypomagnesia, low phosphate, low K, thrombocytopenia, dysrhythmias, kidney failure encourage the child to do low impact exercises – braces and splints for support

scoliosis
lateral curvature of the spine and spinal rotation that causes rib asymmetry diagnosis – bend at the waist with arms hanging while assessing for asymmetry of the rubs and flank treatment – bracing or spinal fusion with rod placement

Cerebral palsy
impairment of motor function, coordination, and posture Abnormal perception and sensation, visual, hearing, and speech impairments; seizures, and cognitive disabilities Cause is unknown – correlated with prenatal risk factors Assessment findings: o Spastic hypertonicity o Dyskinetic (non-spastic, extrapyramidal) – jerking movements that appear slow and wormlike of the trunk neck face and tongue o Ataxic s/s – wide based gait and difficulty with coordination, difficulty with precise movements, and low muscle tone Treatments – skeletal muscle relaxants [Baclofen] + valium [diazepam] Complications – aspiration *, elevated HOB, handle secretions, risk for injury

Spina bifida
failure of the osseous spine to close Neural tube defects are present at birth and effect the CNS and osseous spine Occulta – not visible Cystica – protrusion of the sac is visible Meningocele – contains spinal fluid and the meninges Myelomeningocele – contains spinal fluid, meninges, and nerves Associated with a lack of folate acid during pregnancy s/s: o cystica – protruding sac midline of the spine o occulta – dimpling of the lumbosacral area + port wine angioma + dark hair tufts, subcutaneous lipoma interventions – close asap o sterile moist non-adherent dressing and change Q2 hours o prone position with hips flexed and legs abducted o no pressure on the sac complications – skin ulceration, latex allergies *, increased ICP, bladder issues, and orthopedic issues

down syndrome
chromosomal abnormality small round head, flattened forehead, small nose with depressed nasal bridge, small ears with short pinna, protruding abd, hypotonia and hyper flexibility manage secretions and help prevent respiratory infections cardiac defects and strabismus rinse mouth after feeding and throughout the day cool mist humidification and use bulb syringe prn

juvenile idiopathic arthritis
chronic autoimmune inflammatory disease affecting the joints and other tissues joint swelling, stiffness, redness and warmth worse in morning or after naps apply a splint for sleeping encourage use of a firm mattress and discourage use of pillows apply heat or warm moist packs to the affected joints encourage warm baths NSAIDS, methotrexate, steroids

Muscular dystrophy
group of inherited disorders with progressive degeneration of symmetric skeletal muscle groups causing progressive muscle weakness and wasting Most common Duchenne’s MD – onset within 3-7years s/s: muscle weakness, unsteady gait, waddling, lordosis, and delayed motor skills development o frequent falling, learning difficulties, progressive muscle atrophy resp. and cardiac difficulties around age 20 corticosteroids complications: resp. compromise progressive weakening of the resp. muscles

Impetigo
Caused by staph Reddish macule that becomes vascular and can erupt forming dry crusty’s + itchy Direct contact Abx ointment, burow’s solution

Cellulitis
Firm swollen red area of the skin and subcutaneous tissue Fever Abx and warm moist compresses

Tinea
fungal, Round red scaley patches, itchy in warm and moist areas Head – selenium sulfide shampoo Topical antifungal Treat infected pets as well

Lyme disease
Bit by a tick carrying borrelia burgdorferi Stage 1 – 3-31 days: flu like s/s * bullseye rash at the bite area Stage 2 – after 31 days more systemic issues – paralysis, swelling in joints, weakness Stage 3 – deaf, encephalopathy, arthritis, weakness, numbness and tingling, and speech issues

Scabies
Itchiness, rash, thin pencil mark lines, pimples on trunk, blisters on palms and soles Apply 5% permethrin cream everywhere + family + wash everything in hot water

Pediculosis capitits (lice)
Small red bumps on the scalp, nits (white specks) on the hair shaft Shampoo containing 1% permethrin + remove nits with special comb + wash everything in hot water Can’t wash? Bag for 14 days Boil hair products for 1 hour in lice killing solution

Dermatitis

  • diaper rash Washing with warm water and mild soap Expose to air Encourage parents to use good diapers + frequent changes No bubble baths Skin barrier – zinc oxide containing Corn starch to reduce friction NOT TALCUM POWDER

Poison ivy
Plant exposure – treat area with alcohol followed by water then mild soap and water Then apply a calamine lotion Or a burrow solution Steroid gel

Seborrheic dermatitis
cradle cap Scaly and greasy thick flakes Not contagious Gently scrub the scalp with mild shampoo or special treatment Fine tooth comb Keep nails trimmed short for skin issue kids + gloves or socks over hands for sleeping Cotton clothing Avoid excessive heat Avoid irritants

Atopic dermatitis – eczema
Intense itchy Damage from so much itching Antihistamines + topical steroids

Acne
Good diet, exercise, mild cleanser, don’t pick Meds

Nursing interventions for burns:
ABC’s IV access with lg bore catheters Immunization status – tetanus in last 5 years = they are going to get one Advise family nothing greasy on burn

Fluids based on urine output Kids <30 kg (66 pounds) 1-2ml/kg/hour >30 kg 30ml/hour LR and NS sometimes Manage pain – IV opioid’s Nutrition – increase protein and calories + vitamin A, C, and Zinc Restoring mobility – active and passive ROM Silver sulfadiazine – 2 nd and 3rd degree transient neutropenia Allograft – cavader Xenograft – animals Autograft – own skin

Hypoglycemia s/s: cold and clammy need some candy <60 blood sugar
Hunger Shakiness Diaphoresis Irritability Pale cool skin Possible in LOC Slurred speech, HA, seizures Tachycardia and palpitations Normal to shallow respirations

Hyperglycemia s/s: hot and dry, sugar is high
Polyuria Polydipsia Polyphagia Dehydration Tired/fatigued Weak Nausea, vomiting, abd pain Weak pulse and diminished reflexes Warm, dry, and flushed skin Rapid, deep respiration – kussmauls respirations – fruity smelling breath

Diabetes mellitus diagnostic criteria
8-hour fasting blood glucose of 126 or higher o No antidiabetic meds until after the procedure A random blood glucose of 200 or more + classic s/s of diabetes Oral glucose tolerance test of 200 or more in a 2-hour sample o Balanced diet for 3 days prior o Fast for 8 hours o A fasting level is drawn at the start of the test o Then instructed to consume a specific amount of glucose – and blood levels are drawn every 30 minutes for 2 hours o Assess for hypoglycemia throughout the procedure HbA1c [glycosylated hemoglobin] o Expected range is 4-5.9% but an acceptable range for a child with diabetes can be 6-5-8% with a goal of <7% Less than 7% indicates that DM is being well managed Self-monitored blood glucose – before meals and at bedtime

Foot care of Diabetes Mellitus
Pt. inspects feet daily for wounds Dry feet completely after showers Mild foot powder – corn starch Never use commercial remedies for removing calluses or corns Cut toenails straight across Separate overlapping toes with cotton or lamb’s wool to avoid injury Avoid open toe and heel shoes Leather is preferred to plastic Wear slippers with soles and never go barefoot Check shoes and shake them out Clean absorbent socks made of cotton or wool Not use hot water bottles or heating pads Check water temps with hands, not feet

Management of DM when a pt. is sick
Monitor BG every 3 hours Continue to take insulin or oral meds Encourage sugar free non-caffeinated liquids to prevent dehydration Test urine for ketones Q3 hours Rest Call HCP if BG >240 or with a fever 102 & if ketones are in urine, rapid breathing, or confusion

Hypoglycemic patients [<60]
Treat with 10-15-gram simple carbohydrates [1 table spoon of sugar] o 4 oz. of orange juice, 8 oz. of milk, 3-4 glucose tablets, 4 oz. regular soft drink Unconscious patients glucagon IM or subq + give a simple carb once they wake up

Complications of diabetes mellitus
DKA life threatening condition when BG is over 330 & usually due to an acute illness, non-compliance, or stress o Ketonemia + glycosuria + ketonuria + acidosis [pH 7.30 and bicarb 15] resulting in the breakdown of body fat for energy and an accumulation of ketones in the blood, urine, and lungs o Rapid onset o Fruity breath, deep breathing, kussmauls, confusion, dyspnea, n/v, dehydration, and electrolyte imbalances o Metabolic acidosis – hyperkalemia o Treatment – as we are bringing the glucose levels down, the potassium levels may switch from hyper to hypo Cardiac monitor Sodium bicarb for metabolic acidosis slow IV infusion When BG levels get around 250 – add glucose to IV fluids in order to maintain 120-240 BG Give IV insulin continuously Monitor levels hourly

growth hormone deficiency
short stature, delayed bone closure, and delayed sex development

growth hormone treatment

  • somatropin subcutaneous injections until the bones have closed

Immunizations
Common cold or minor illness – not contraindications for getting vaccinated Severe acute illness – contraindicated Immunosuppression – contraindicated from a few Flu vaccine – hypersensitivity to eggs = c/a Varicella – corticosteroids = c/a IPV – allergy to neomycin = c/a MMR – allergy to gelatin & neomycin = c/a DTAP – occurrence of encephalopy, seizures, or inconsolable crying that lasted a long time previously = c/a VL or ventral gluteal for smaller children Older children – deltoid muscle Charting include date, route, site, type, manufacture lot number, and expiration Low grade fever (common s/e) – don’t give aspirin = Reyes syndrome Babies vaccine can give concentrated oral sucrose solution on a pacifier 2 min before and for 3 min after the injection

A nurse is reinforcing home safety instructions with the parents of a toddler. Which of the following parent statements indicates an understanding of the teaching?
“We will turn the pot handles toward the back of the stove.”
The nurse should instruct the parents to turn pot handles toward the back of the stove to prevent the toddler from pulling a pot off the stove, resulting in a burn.

A nurse is caring for an adolescent client who is a practicing Jehovah’s Witness and is scheduled for surgery for a ruptured appendix. The adolescent tells the nurse that based on her religious beliefs, she cannot receive a blood transfusion. Which of the following responses should the nurse make?
“Let’s discuss the possibility of you needing a blood transfusion with your parents.”
The nurse should offer to involve the child’s parents to understand the family’s beliefs about blood transfusions.

A nurse is reviewing the laboratory report of a preschooler. Which of the following laboratory results should the nurse report to the provider?
Lead 14 mcg/dL
This lead level is above the expected reference range for a preschooler. Therefore, the nurse should report this result to the provider.

A nurse is collecting data from a 12-month-old infant during a well-child visit. The nurse should identify which of the following findings as a deviation from expected growth and development?
Birth weight doubled
The nurse should identify this finding as a deviation from expected growth and development. The infant’s birth weight should triple by 12 months of age. Therefore, the nurse should report this finding to the provider.

A nurse is reinforcing discharge teaching with the guardian of a school-age child who has acute lymphocytic leukemia and an absolute neutrophil count of 450/mm3. Which of the following instructions should the nurse include?
“Keep your child away from crowded areas.”
The nurse should instruct the guardian to keep the child away from crowds and visitors who have an illness to decrease the risk for infection.

A nurse in a clinic is collecting data from an adolescent who has received all recommended immunizations through the age of 6 years. Which of the following immunizations should the nurse plan to administer?
Tetanus, diphtheria toxoids, and acellular pertussis (Tdap)
The Tdap vaccine is recommended between the ages of 11 and 12 years. Therefore, this adolescent should receive the Tdap vaccine now.

A nurse is reinforcing teaching with the parent of a child who has a new prescription for ferrous sulfate. The nurse should reinforce that the parent should administer the medication with which of the following fluids to enhance the medication absorption?
Orange juice
The nurse should reinforce with the parent that administering ferrous sulfate with orange juice will enhance medication absorption.

A nurse in a pediatric clinic is observing for an anaphylactic reaction after administering an IM antibiotic to a child 5 min ago. Which of the following manifestations should the nurse expect to observe first?
Hives
The nurse should observe for hives first because this is an early manifestation of an anaphylactic reaction.

A nurse is reinforcing dietary teaching about a low-sodium diet with the parents of a child who is recovering from acute glomerulonephritis. Which of the following food choices by the parents indicates an understanding of the teaching?
Apples
The nurse should instruct the parents that apples are low in sodium and supply the child with energy needed for recovery.

A nurse is collecting data from a 12-month-old infant during a well-child visit. Which of the following findings should the nurse report to the provider?
BP 115/70 mm Hg
The nurse should identify that this blood pressure is above the expected reference range for a 12-month-old infant and report this finding to the provider.

A nurse is reinforcing discharge teaching with the guardian of a child who has juvenile idiopathic arthritis (JIA). Which of the following statements by the parent indicates an understanding of the teaching?
“I will have my child sleep in knee, wrist, and hand splints.”
The nurse should reinforce with the guardian that splinting the child’s joints at night will decrease pain and enhance joint function.

A nurse is collecting data about the dietary habits of an adolescent client. The nurse should identify that which of the following findings puts the client at risk for nutritional deficits?
The client skips eating dinner for track practice three times per week.
The nurse should identify that adolescents are often at risk for developing poor eating habits. Skipping dinner twice each week puts this client at risk for nutritional deficits.

A nurse is reinforcing teaching with the guardians of a school-age child who has hearing loss. Which of the following techniques should the nurse recommend to facilitate communication with the child?
Speak at the child’s eye level.
The nurse should instruct the guardian to speak at the child’s eye level and ensure that there is adequate lighting on the speaker’s face to facilitate lipreading and communication.

A nurse is assisting with a sterile dressing change for an adolescent who has a partial thickness burn on the right hip. Which of the following actions should the nurse take first?
Administer pain medication to the client.
According to evidence-based practice, the nurse should first provide pain medication to the client to reduce discomfort during the procedure.

A nurse is reviewing the laboratory report of a school-age child who is receiving prednisone. Which of the following laboratory results should the nurse report to the provider?
Sodium 150 mEq/L
Hypernatremia is an adverse effect of prednisone. This level is above the expected reference range for a school-age child. Therefore, the nurse should report this value to the provider.

A nurse in a community center is reinforcing teaching about poison control with a group of parents. A parent asks what to do if a child ingests a large quantity of acetaminophen. Identify the sequence of actions the nurse should recommend to the parent.
Determine if the child is breathing.
Empty the child’s mouth of remaining pills and residue.
Identify the medication and dosage strength.
Call a poison control center.
The child’s respiratory and cardiovascular status should be checked first to determine if CPR is necessary. Then, the child’s mouth should be emptied of pills and residue to prevent additional exposure to the medication. Next, the parent should identify the medication and dosage strength by looking at the medication container. Lastly, the parent should contact a poison control center for advice on the next course of action.

A nurse is caring for a preschooler who has a new diagnosis of asthma. Which of the following medications should the nurse instruct the parent to administer for an acute asthma attack?
Albuterol
The nurse should inform the parent to administer albuterol, a short-acting beta2 agonist, to the preschooler for acute asthma attacks.

A nurse is assisting with the care of an infant who has spina bifida and recently had a ventriculoperitoneal shunt placed for hydrocephalus. Which of the following findings should the nurse identify as an indication of increased ICP?
High-pitched cry
The nurse should identify that a high-pitched cry is an indication of increased intracranial pressure.

A nurse is reinforcing discharge teaching with the guardian of a school-age child who has a new prescription for home oxygen therapy. Which of the following statements by the guardian indicates an understanding of the teaching?
“I will make sure that electrical devices in the house are grounded.”
This response by the guardian indicates an understanding of the nurse’s instructions. Due to the combustible nature of oxygen, all pieces of electrical equipment in the home should be grounded to decrease the risk of a fire caused by an electrical spark.

A nurse is caring for a school-age girl who is being treated for frequent, severe UTI’s. The nurse should recognize that which of the following statements by the parent indicates a possible cause of the UTI’s?
“My daughter has bowel movements every 4 to 5 days.”
The nurse should recognize that this frequency indicates the child is constipated. Therefore, large stool masses might prevent complete emptying of the bladder and lead to urinary stasis and infection.

A nurse is preparing to administer the measles, mumps, and rubella (MMR) vaccine to a preschooler. The nurse should recognize which of the following statements by the parent as a contraindication to receiving the immunization?
“My child received an immunoglobulin last month.”
The nurse should identify that a preschooler who received an immunoglobulin less than 1 month ago should not receive the MMR vaccine on this day. The nurse should instruct the parent to reschedule the immunization after 3 months have elapsed, since the child received passive immunity via administration of an immunoglobulin.

A nurse is reinforcing teaching with the guardian of a child who has scabies and a new prescription for permethrin 5% cream. Which of the following information should the nurse include?
“The medication will eliminate your child’s itching within 2 to 3 weeks.”
The nurse should instruct the guardian that, although the medication kills the mites, itching can continue for 2 to 3 weeks following application of the medication.

A nurse is collecting data from an infant who has severe dehydration. Which of the following findings should the nurse expect?
Weight loss of 10%
The nurse should expect an infant who has severe dehydration to experience weight loss of 10% or greater.

A nurse is assisting with the administration of a nasogastric enteral feeding for an infant. Which of the following actions should the nurse take?
Place the infant in semi-Fowler’s position for 1 hr after the feeding.
The nurse should elevate the head of the infant’s bed by 30º to 45º for 30 min to 1 hr after the feeding.

A nurse is caring for a school-age child who has been admitted to the facility in sickle cell crisis. The nurse is measuring the child’s oral intake for the shift. The child consumed 4 oz of juice at breakfast. For lunch, the child consumed 6 oz of milk, 6 oz of gelatin, and drank 7 oz of water. What is the child’s oral intake for this shift in mL?
690 mL
1 oz = 30 mL

A nurse is assisting with the care of a 3-year-old child who is prescribed a lumbar puncture. Which of the following actions should the nurse take to prevent complications?
Maintain the child in a flat position after the procedure.
After a lumbar puncture, the optimal position for the client is flat and supine to prevent headaches.

A nurse is collecting data from a 6-month-old child who is experiencing a sickle cell crisis. Which of the following areas should the nurse observe when monitoring for manifestations of splenic sequestration?
The nurse should observe the location over the infant’s spleen (LUQ of abdomen) when monitoring for manifestations of splenic sequestration. Splenic sequestration is an enlargement of the spleen due to pooling of sickled cells in the blood.

A nurse is reinforcing teaching with an adolescent female client who has acne vulgaris and a new prescription for isotretinoin. Which of the following information should the nurse include?
“You will need to have two negative pregnancy tests prior to starting this medication.”
The nurse should reinforce with the client that isotretinoin is teratogenic. Pregnancy must be ruled out prior to administration and before each subsequent refill. The client should use two effective forms of contraception while taking this medication.

A nurse is assisting the provider with a developmental assessment of a toddler. Which of the following behaviors should the nurse recognize as an expected finding?
Stands on one foot for several seconds
Standing on one foot for several seconds is an expected behavior for a toddler.

A nurse is reinforcing teaching about home safety with the parent of a toddler. Which of the following parent statements indicates an understanding of the teaching?
“I will place a screen in front of the fireplace.”
The nurse should instruct the parent to place a screen in front of a fireplace or other heating appliances to prevent burns.

A nurse is reinforcing dietary teaching with the guardian of a school-age child who has celiac disease. Which of the following foods should the nurse recommend including in the child’s diet?
White rice
The nurse should reinforce to the guardian that celiac disease is a genetic autoimmune disorder in which eating gluten, even in very small amounts, can damage the child’s small intestine. Currently, the only treatment for the disease is a lifelong, strict adherence to a gluten-free diet. The nurse should stress the importance of avoiding foods containing wheat, rye, barley, and oats. The child should consume foods that are gluten-free, such as milk, cheese, rice, corn, eggs, potatoes, fruits, vegetables, fresh poultry, meats, fish, and dried beans.

A nurse is collecting data from a toddler who has gastroesophageal reflex disease (GERD). Which of the following findings should the nurse expect?
Chronic cough
The nurse should identify that a chronic cough is an expected finding in a child who has GERD.

A nurse is reinforcing teaching about injury prevention with the guardian of an infant. Which of the following statements by the guardian indicates an understanding of the teaching?
“I should make sure my baby’s clothing does not have buttons on it.”
The nurse should instruct the guardian to avoid dressing the infant in clothing with buttons to reduce the risk of choking and aspiration.

A nurse is reviewing the medical record of a female adolescent client who has primary amenorrhea. Which of the following findings should the nurse identify as a risk factor for this disorder? (Select all that apply.)
Hypothyroidism
The nurse should identify that hypothyroidism and other endocrine disorders are risk factors for primary amenorrhea.
Cannabis use
The nurse should identify that cannabis use is a risk factor for primary amenorrhea.
Oral contraceptive use
The nurse should identify that oral contraceptive use affects the estrogen and progesterone cycle and is a risk factor for primary amenorrhea.
Emotional stress
The nurse should identify that emotional stress causes hypothalamic suppression and is a risk factor for primary amenorrhea.

A nurse is reinforcing teaching with the family of a preschooler whose parent has a terminal diagnosis. Which of the following statements should the nurse include when discussing age-appropriate responses to death?
“At this age, your child likely believes his thoughts can cause another person’s death.”
The nurse should reinforce that, at this age, the preschooler might believe that his thoughts can cause another person’s death, which can make him feel guilty or responsible for the death.

A nurse is reinforcing teaching regarding the immunization schedule of a newborn. Which of the following statements made by the parent should the nurse recognize as an understanding of the newborn’s immunization schedule?
“My baby will receive his next immunization when he is 2 months old.”
Newborns should receive the next scheduled immunization 2 months after birth.

A nurse is preparing to administer an enteral feeding to a child who has cerebral palsy and a nasogastric tube. Which of the following actions should the nurse take?
Confirm that the pH of the stomach contents is 5 or less.
The nurse should test the pH of the stomach contents prior to administering the tube feeding in order to confirm tube placement in the stomach. The nurse should identify that a pH of 5 or less indicates gastric placement.

A nurse is assisting with scoliosis screenings for a group of school-age children. The nurse should place the students in which of the following positions during the screening?
Bending forward with back parallel to the floor
The nurse should observe for asymmetry and prominence of the rib cage by having the students bend forward with the back parallel to the floor.

A nurse is reviewing the laboratory report of a preschooler who has a Wilm’s tumor and is scheduled to begin treatment with an antineoplastic medication regimen. Which of the following laboratory results should the nurse report to the provider?
Platelet count 70,000/mm3
This platelet count is below the expected reference range for a preschooler and increases the risk for spontaneous bleeding. The nurse should hold the medication and report this finding to the provider immediately.

A nurse in a pediatric clinic is collecting data from an infant who recently started taking digoxin. Which of the following manifestations should the nurse identify as an indication of digoxin toxicity and report to the provider?
Vomiting
The nurse should identify that vomiting, especially unrelated to feedings, is a manifestation of digoxin toxicity and should be reported to the provider.

A nurse is reinforcing teaching with the parents of a 7-year-old female child about behavioral expectations. Which of the following behaviors should the nurse include in the teaching?
Spends a lot of time by herself
Spending time alone is an expected characteristic of a 7-year-old female child. When they do spend time with others, children in this age group prefer to socialize with children of the same sex and age.

A nurse is contributing to the plan of care for a child who has sickle cell anemia and is experiencing a vaso-occlusive crisis. Which of the following is the priority intervention for the nurse to recommend to include in the plan?
Promote oxygen utilization
The priority action the nurse should take when using the airway, breathing, circulation (ABC) approach to client care is promoting oxygen utilization to prevent further sickling of the red blood cells and promote adequate oxygenation of the tissue.

A nurse is contributing to the plan of care for an adolescent client who has human immunodeficiency virus (HIV). Based on the adolescent’s diagnosis, which of the following actions should be included in the plan of care?
Inform the client regarding routes of transmission.
The nurse should inform the client about the transmission of HIV and how to prevent its spread.

A nurse is reinforcing teaching with the guardian of a child who has a new diagnosis of enterobiasis. The nurse should advise the guardian to take which of the following actions to prevent infection?
Trim the child’s fingernails short.
The nurse should instruct the guardian to trim the child’s fingernails short to reduce the collection of eggs under her nails and prevent reinfection.

A nurse is collecting data about a 4-year-old preschooler’s gross motor skills. The nurse should expect the preschooler to be able to perform which of the following activities?
Hopping on one foot
The nurse should expect to find that a 4-year-old preschooler is able to hop on one foot.

A nurse is reinforcing discharge teaching with the parent of a school-age child who is being treated for nephrotic syndrome. The parent asks the nurse why it is necessary to check the child’s urine for protein. Which of the following explanations should the nurse offer?
“A decrease in urine protein indicates that treatment is effective.”
The desired outcome of steroid therapy in the treatment of nephrotic syndrome is a reduction of proteinuria.

A nurse is reinforcing teaching about glucose monitoring with the parent of a child who has type 1 diabetes mellitus. Which of the following instructions should the nurse include in the teaching?
“Put your child’s finger under warm, running water prior to collecting blood.”
The nurse should instruct the parent that placing the child’s finger under warm, running water increases the blood flow to the finger, which will make it easier to obtain the sample.

A nurse in a pediatric clinic is collecting data from an infant who was recently exposed to pertussis. The nurse should recognize which of the following as a manifestation of pertussis?
Dry cough
The nurse should identify that a dry cough is an early manifestation of pertussis.

A nurse is reinforcing teaching with the parent of a child who has hemophilia and is experiencing acute hemarthrosis. Which of the following instructions should the nurse include in the teaching?
Keep the affected joints immobilized
The nurse should reinforce with the parent to keep the child’s affected joints elevated and immobilized to minimize bleeding. After the acute episode, the child should begin active range-of-motion exercises.

A nurse is auscultating heart sounds on an infant. The nurse should identify this sound as which of the following? (Audio clip)
Sinus rhythm
The nurse should identify this heart sound as sinus rhythm. The nurse should auscultate heart sounds at the apical impulse, which is at the left midclavicular line and fifth intercostal space. The expected heart sounds include S1, which is the closure of the atrioventricular valves, and S2, which is the closure of the semilunar valves.

A nurse is preparing to administer ophthalmic drops to a child. Which of the following actions should the nurse take?
Apply pressure to the lacrimal punctum for 1 min following administration.
The nurse should apply pressure to the lacrimal punctum to prevent the medication from entering the nasopharynx.

A nurse has just received change-of-shift report for four children in a pediatric unit. Which of the following children should the nurse collect data from first?
A child who has a fever and nuchal rigidity
A client who has a fever and nuchal rigidity is unstable. This finding indicates bacterial meningitis, which requires urgent data collection and intervention to reduce complications for the child and prevent further spread of the infection. Therefore, the nurse should collect data from this child first.

A nurse is caring for a school-age child who has skeletal traction applied to the right lower leg to repair a femur fracture. Which of the following findings is the priority for the nurse to report to the provider?
Report of tingling in the right foot
The nurse should identify that the greatest risk to the child is nerve injury. Therefore, tingling in the right foot, which can indicate nerve damage or compartment syndrome, is the priority finding for the nurse to report to the provider.

A nurse is assisting with the care of a child who has tonic-clonic seizures. Which of the following actions should the nurse take?
Have a suction canister and tubing available in the room.
The nurse should have a suction canister and tubing available in the child’s room to keep the child’s airway patent during a seizure.

A nurse is monitoring a preschooler following an abdominal CT scan with contrast dye. The nurse should identify which of the following as an indication that the preschooler experienced an allergic reaction to the contrast dye?
Urticaria
The nurse should monitor the child for an allergic reaction to the contrast dye. Manifestations of the allergic reaction include urticaria, itching, flushing of the skin, and possible anaphylaxis.

A nurse is collecting data from a child who has iron deficiency anemia. Which of the following data signifies that adherence to ferrous sulfate therapy has occured?
Green, tarry stools
Green, tarry stools are an expected outcome of ferrous sulfate therapy. Therefore, this is an indication of adherence to the prescribed medication regimen.

A nurse is reinforcing teaching with the parents of a child who has cystic fibrosis and is taking pancrelipase as a pancreatic enzyme replacement. The nurse should plan to inform the child’s parents that the therapeutic effects of this medication can be evaluated by which of the following?
Amount and consistency of stools
Recording the amount and consistency of the child’s stools will help determine the effectiveness of pancrelipase, which is taken to decrease the bulk of feces.

A nurse is reinforcing teaching with the parents of a toddler who has strabismus. Which of the following treatments should the nurse plan to include in the teaching?
Eye patch
Treatment of strabismus includes covering the strong eye to strengthen the muscles in the weak eye.

A nurse is reinforcing teaching about tracheostomy care with the parent of a toddler who has a temporary tracheostomy. Which of the following instructions should the nurse include in the teaching?
Ensure one finger fits between the ties and the neck.
The nurse should instruct the parent that one finger should fit between the ties and the neck to ensure the tube is held securely in place.

A nurse is collecting data from a toddler at a well-child visit. Which of the following findings should the nurse identify as a possible indication of child maltreatment?
Laceration on the side of the torso
A laceration on the side of the torso is not an injury that occurs due to the typical clumsiness of a toddler. This finding indicates the need to further investigate for suspected child maltreatment.

A nurse is creating a plan of care for a school-age child who has heart disease and has developed heart failure. Which of the following interventions should the nurse include in the plan?
Provide small, frequent meals for the child.

The metabolic rate of a child who has heart failure is high because of poor cardiac function. Therefore, the nurse should provide small, frequent meals for the child because it helps to conserve energy.

A nurse is teaching the parent of an infant who has a Pavlik harness for the treatment of developmental dysplasia of the hip. The nurse should identify that which of the following statements by the parent indicates an understanding of the teaching?
“I will place my infant’s diapers under the harness straps.”

To prevent soiling of the harness, the parent should apply the infant’s diaper under the straps.

A nurse is planning care for a school-age child who is in the oliguric phase of acute kidney injury (AKI) and has a sodium level of 129 mEq/L. Which of the following interventions should the nurse include in the plan?
Initiate seizure precautions for the child.

A sodium level of 129 mEq/L indicates hyponatremia and places the child at increased risk for neurological deficits and seizure activity. The nurse should complete a neurologic assessment and implement seizure precautions to maintain the child’s safety.

A nurse is assessing a school-age child immediately following a perforated appendix repair. Which of the following findings should the nurse expect?
Absence of peristalsis

The nurse should expect absence of peristalsis immediately following a perforated appendix repair, until the bowel resumes functioning.

A nurse is preparing an adolescent for a lumbar puncture. Which of the following actions should the nurse take?
Apply topical analgesic cream to the site 1 hr prior to the procedure.

The nurse should apply a topical analgesic to the lumbar site 1 hr prior to the procedure to decrease the adolescent’s pain while the lumbar needle is inserted.

A nurse is caring for a school-age child who is receiving cefazolin via intermittent IV bolus. The child suddenly develops diffuse flushing of the skin and angioedema. After discontinuing the medication infusion, which of the following medications should the nurse administer first?
Epinephrine

This child is most likely experiencing an anaphylactic reaction to the cefazolin. According to evidence-based practice, the nurse should first administer epinephrine to treat the anaphylaxis. Epinephrine is a beta adrenergic agonist that stimulates the heart, causes vasoconstriction of blood vessels in the skin and mucous membranes, and triggers bronchodilation in the lungs.

A nurse is teaching the parent of a preschooler about ways to prevent acute asthma attacks. Which of the following statements by the parent indicates an understanding of the teaching?
“I should keep my child indoors when I mow the yard.”

The nurse should instruct the parent to keep the preschooler indoors during lawn maintenance or when the pollen count is increased. Guarding against exposure to known allergens found outdoors, such as grass, tree, and weed pollen, will decrease the frequency of the preschooler’s asthma attacks.

A nurse is proving dietary teaching to the parent of a school-age child who has celiac disease. The nurse should recommend that the parent offer which of the following foods to the child?
White rice

The nurse should recommend that the parent offer white rice to the child because it is a gluten-free food. The nurse should instruct the parent that the child will remain on a lifelong gluten-free diet and the child should not consume oats, rye, barley, or wheat, and sometimes lactose deficiency can be secondary to this disease.

A nurse is reviewing the laboratory report of a school-age child who is experiencing fatigue. Which of the following findings should the nurse recognize as an indication of anemia?
Hematocrit 28%

The nurse should recognize that this hematocrit level is below the expected reference range of 32% to 44% for a school-age child. The child can exhibit fatigue, lightheadedness, tachycardia, dyspnea, and pallor due to the decreased oxygen-carrying capacity.

A nurse is preparing to collect a sample from a toddler for a sickle-turbidity test. Which of the following actions should the nurse plan to take?
Perform a finger stick.

The nurse should perform a finger stick on a toddler as a component of the sickle-turbidity test. If the test is positive, hemoglobin electrophoresis is required to distinguish between children who have the genetic trait and children who have the disease.

A nurse is assessing a school-age child who has meningitis. Which of the following findings is the priority for the nurse to report to the provider?
Petechiae on the lower extremities

The presence of a petechial or purpuric rash on a child who is ill can indicate the presence of meningococcemia. This type of rash indicates the greatest risk of serious rapid complications from sepsis and should be reported immediately to the provider.

A nurse is assessing an infant who has a ventricular septal defect. Which of the following findings should the nurse expect?
Loud, harsh murmur

The nurse should expect to hear a loud, harsh murmur with a ventricular septal defect due to the left-to-right shunting of blood, which contributes to hypertrophy of the infant’s heart muscle.

A nurse is creating a plan of care for an infant who has an epidural hematoma from a head injury. Which of the following interventions should the nurse include in the plan?
Implement seizure precautions for the infant.

An infant who has an epidural hematoma is at great risk for seizure activity. Therefore, the nurse should implement seizure precautions for the child.

A nurse is caring for an adolescent who received a kidney transplant. Which of the following findings should the nurse identify as an indication the adolescent is rejecting the kidney?
Serum creatinine 3.0 mg/dL

Creatinine is a byproduct of protein metabolism and is excreted from the body through the kidneys. An elevated serum creatinine level, therefore, can be an indication that the kidneys are not functioning. The nurse should identify that the adolescent’s serum creatinine level is higher than the expected reference range of 0.4 to 1.0 mg/dL for an adolescent and can indicate rejection of the kidney.

A nurse in an emergency department is performing an admission assessment on a 2 week-old male newborn. Which of the following findings is the priority for the nurse to report to the provider?
Substernal retractions

When using the airway, breathing, and circulation approach to client care, the nurse should determine that the priority finding to report to the provider is substernal retractions. This finding indicates the newborn is experiencing increased respiratory effort, which could quickly progress to respiratory failure.

A hospice nurse is caring for a preschooler who has a terminal illness. The father tells the nurse that he cannot cope anymore and has decided to move out of the house. Which of the following statements should the nurse make?
“Let’s talk about some of the ways you have handled previous stressors in your life.”

This statement offers a general lead to allow the parent to express their feelings and previous actions when faced with stressful situations. It also helps the parent to focus on ways that they can cope with the current situation.

A nurse in an emergency department is caring for an adolescent who has severe abdominal pain due to appendicitis. Which of the following locations should the nurse identify as McBurney’s point?
A. The nurse should identify this area of the client’s abdomen as McBurney’s point. This area of the right lower quadrant located about two-thirds of the way between the umbilicus and the client’s anterosuperior iliac spine is the area where a client who has appendicitis is most likely to report pain and tenderness.

A nurse is reviewing the laboratory report of a 7 year-old child who is receiving chemotherapy. Which of the following lab values should the nurse report to the provider?
Hgb 8.5 g/dL

A child receiving chemotherapy is at risk for anemia due to the chemotherapy effects on the blood-forming cells of the bone marrow. The development of anemia is diagnosed through laboratory testing of hemoglobin and hematocrit levels. The nurse should recognize that a hemoglobin level of 8.5 g/dL is below the expected reference range of 10 to 15.5 g/dL for a 7-year-old child and should be reported to the provider.

A nurse is caring for a 15 year-old client who is married and is scheduled for a surgical procedure. The client asks, “who should sign my surgical consent?” Which of the following responses should the nurse make?
“You can sign the consent form because you are married.”

The nurse should inform the adolescent that marriage gives adolescents the legal right to consent to surgical procedures and sign other legal documents that they would not otherwise be able to sign due to their age.

A nurse is assessing a 4-year-old child at a well-child visit. Which of the following developmental milestones should the nurse expect to observe?
Cuts an outlined shape using scissors.

The nurse should recognize that an expected developmental milestone of a 4-year-old child is using scissors to cut out a shape.

A nurse is caring for an infant who has respiratory syncytial virus (RSV). Which of the following actions should the nurse implement for infection control?
Have a designated stethoscope in the infant’s room.

The nurse should initiate droplet precautions for an infant who has RSV because the virus is spread by direct contact with respiratory secretions. Therefore, designated equipment, such as a blood pressure cuff and a stethoscope, should be placed in the infant’s room.

A nurse in an emergency department is caring for a school-age child who has appendicitis and rates their abdominal pain as 7 on a scale of 0 to 10. Which of the following actions should the nurse take?
Give morphine 0.05mg/kg IV

A pain level of 7 on a scale of 0 to 10 is considered severe. The nurse should administer an analgesic medication for pain relief.

A nurse is assessing the vital signs of a 10-year-old child following a burn injury. The nurse should identify that which of the following findings in an indication of early septic shock?
Temperature 39.1° C (102.4° F)

The nurse should identify that a temperature of 39.1° C (102.4° F) is above the expected reference range of 37° to 37.5° C (98.6° to 99.5° F) for a 10-year-old child. The nurse should expect a child who has early septic shock to have a fever and chills.

A school nurse is assessing an adolescent who has multiple burns in various stages of healing. Which of the following behaviors should the nurse identify as a possible indication of physical abuse?
Denies discomfort during assessment of injuries.

The nurse should suspect child maltreatment in the form of physical abuse if the adolescent has a blunted response to painful stimuli or injury.

A nurse is caring for a 15 year-old client following a head injury. Which of the following findings should the nurse identify as an indication that the child is developing syndrome of inappropriate antidiuretic hormone secretion (SIADH)?
Mental confusion

A child who has a head injury can develop SIADH as a result of altered pituitary function, leading to an oversecretion of antidiuretic hormone. Oversecretion of antidiuretic hormone leads to a decrease in urine output, hyponatremia, and hypoosmolality due to overhydration. As the hyponatremia becomes more severe, mental confusion and other neurologic manifestations such as seizures can occur.

A nurse is caring for a toddler who has spastic (pyramidal) cerebral palsy. Which of the following findings should the nurse expect? (Select all that apply.)
-Ankle clonus
-Exaggerated stretch reflexes
-Contractures

A nurse in a provider’s office if preparing to administer immunizations to a toddler during a well-child visit. Which of the following actions should the nurse plan to take?
Withhold the measles, mumps, and rubella (MMR) vaccine.

The nurse should recognize that an allergy to neomycin with an anaphylactic reaction is a contraindication for receiving the MMR vaccine. Clients who have a severe allergy to eggs or gelatin should not receive this vaccine.

A school nurse is assessing an adolescent who has scoliosis. Which of the following findings should the nurse expect?
A unilateral rib hump

When assessing an adolescent for scoliosis, the school nurse should expect to see a unilateral rib hump with hip flexion. This results from a lateral S- or C-shaped curvature to the thoracic spine resulting in asymmetry of the ribs, shoulders, hips, or pelvis. Scoliosis can be the result of a neuromuscular or connective tissue disorder, or it can be congenital in nature.

A nurse is caring for a preschooler whose father is going home for a few hours while another relative stays with the child. Which of the following statements should the nurse make to explain to the child when their father will return?
“Your daddy will be back after you eat.”

Preschoolers make sense of time best when they can associate it with an expected daily routine, such as meals and bedtime. Therefore, the child comprehends time best when it is explained to them in relation to an event they are familiar with, such as eating.

The nurse is caring for a preschooler who has been receiving IV fluids via a peripheral IV catheter. When preparing to discontinue the IV fluids and catheter, which of the following actions should the nurse plan to take?
First, the nurse should turn off the IV pump. Next, the nurse should occlude the IV tubing, and then remove the tape securing the catheter. Last, the nurse should apply pressure over the catheter insertion site.

A nurse is caring for a school-age child who has experienced a tonic-clonic seizure. Which of the following actions should the nurse take during the immediate postictal period?
Place the child in a side-lying position.

The nurse should place the child in a side-lying position to prevent aspiration.

A nurse is teaching the guardian of a 6-month-old infant about car seat use. Which of the following statements by the guardian indicates an understanding of the teaching?
“I should secure the car seat using lower anchors and tethers instead of the seat belt.”

Lower anchors and tethers, or the LATCH child safety seat system, should be used to secure an infant’s car seat in the vehicle. This system provides anchors between the front cushion and the back rest for the car seat. Therefore, if this system is available, the seat belt does not have to be used.

A nurse in an urgent care clinic is assessing an adolescent who has an upper respiratory tract infection. Which of the following findings should the nurse identify as a manifestation of pertussis?
Dry, hacking cough

The nurse should identify that a dry, hacking cough is a manifestation of pertussis. This disease usually begins with indications of an upper respiratory tract infection, which includes a dry, hacking cough that is sometimes more severe at night.

A nurse is preparing to administer an immunization to a 4-year-old child. Which of the following actions should the nurse plan to take?
Administer the immunization using a 24-gauge needle.

The nurse should administer an immunization for a 4-year-old child using a 22- to 25- gauge needle to minimize the amount of pain the child experiences.

The nurse is providing discharge teaching to the parent of a child who is 1 week postoperative following a cleft palate repair. For which of the following members of the inter professional team should the nurse initiate a referral?
Speech therapist

The nurse should initiate a referral for a speech therapist for a child who is postoperative following a cleft palate repair. A child who has a cleft palate will require speech therapy immediately following the repair to support speech development and future articulation.

A school nurse is preparing to administer atomoxetine 1.2 mg/kg/day PO to a school aged child who weights 75 lbs. Available is atomoxetine 40 mg/capsule. How many capsules should the nurse administer per day?
1 capsule

A nurse is caring for a school-age child who is receiving a blood transfusion. Which of the following manifestations should alert the nurse to a possible hemolytic transfusion reaction?
Flank pain

The nurse should recognize that flank pain is caused by the breakdown of RBCs and is an indication of a hemolytic reaction to the blood transfusion.

A nurse in the emergency department is caring for a toddler who has a partial thickness burns on their right arm. Which of the following actions should the nurse take?
Cleanse the affected area with mild soap and water.

The nurse should wash the affected area with mild soap and water to remove any loose tissue that could cause infection.

A nurse is auscultating the lungs of an adolescent who has asthma. The nurse should identify the sound as which of the following?
Tachypnea

The nurse should identify the sound heard during auscultation as tachypnea, which is a rapid, regular breathing pattern. This breathing pattern often occurs with anxiety, fever, metabolic acidosis, or severe anemia.

A nurse is reviewing the lumbar puncture results of a school-age child suspected of having bacterial meningitis. Which of the following results should the nurse identify as a finding associated with bacterial meningitis?
Increased protein concentration

The nurse should identify that an increased protein concentration in the spinal fluid is a finding that can indicate bacterial meningitis.

A nurse is assessing a 3-year-old toddler at a well-child visit. Which of the following manifestations should the nurse report to the provider?
Respiratory rate 45/min

The nurse should identify that a respiratory rate of 45/min is above the expected reference range of 20 to 25/min for a 3-year-old toddler and can indicate respiratory dysfunction and acute respiratory distress. Therefore, the nurse should report this finding to the provider.

A nurse is teaching the parents of a school-age child who has a new diagnosis of osteomyelitis of the tibia. The nurse should identify that which of the following statements by the parents indicates an understanding of the teaching?
“My child will receive antibiotics for several weeks.”

The nurse should instruct the parent that the child will receive antibiotic therapy for at least 4 weeks. Surgery might be indicated if the antibiotics are not successful.

A nurse is providing teaching about social development to the parents of a preschooler. Which of the following play activities should the nurse recommend for the child?
Playing dress-up

The nurse should instruct the parents that at the preschool age, play should focus on social, mental, and physical development. Therefore, playing dress-up is a recommended play activity for this child.

A nurse is reviewing the laboratory report of an infant who is receiving treatment for severe dehydration. The nurse should identify that which of the following laboratory values indicates effectiveness of the current treatment?
Sodium 140 mEq/L

The nurse should identify that a sodium level of 140 mEq/L is within the expected reference range of 134 to 150 mEq/L and indicates the current treatment regimen the infant is receiving for dehydration is effective.

A nurse is caring for a preschooler who is scheduled for hydrotherapy treatment for wound debridement following a burn injury. Which of the following actions should the nurse take prior to the procedure?
Administer an analgesic to the child.

Hydrotherapy for debridement of a wound is an extremely painful procedure which requires analgesia and/or sedation. When pain is controlled, it leads to reduced physiological demands on the body caused by stress and decreases the likelihood of children developing depression and post-traumatic stress disorder.

A charge nurse in an emergency department is preparing an in-service for a group of newly licensed nurses on the clinical manifestations of child maltreatment. Which of the following manifestations should the charge nurse include as suggestive of potential physical abuse?
Symmetric burns of the lower extremities

The nurse should include that symmetric burns to the lower extremities can indicate physical abuse. The patterns are usually characteristic of the method or object used, such as cigar or cigarette burns, or burns in the shape of an iron.

A nurse is caring for a school-age child who in in Buck’s traction following a leg fracture 24 hr ago. Which of the following actions should the nurse take?
Assess peripheral pulses once every 4 hr.

Buck’s traction is a type of skin traction that can be used to immobilize extremities prior to surgery. The nurse should provide frequent neurovascular checks at least every 4 hr after the first 24 hr of placement in Buck’s traction. The nurse should monitor and report signs of neurovascular impairment in the extremities such as cyanosis, edema, pain, absent pulses, and tingling.

A nurse is planning care for a toddler who has a serum lead level of 4 mcg/dL. Which of the following actions should the nurse plan to take?
Schedule the toddler for a yearly rescreening.

The nurse should schedule the toddler for a lead level rescreening in 1 year and educate the family on ways to prevent exposure.

A nurse is receiving change-of-shift report on four children. Which of the following children should the nurse see first?
A school-age child who has sickle cell anemia and reports decreased vision in the left eye.

When using the urgent vs. nonurgent approach to client care, the nurse should determine the priority finding is a report of decreased vision in the left eye. This finding indicates that the child is experiencing a vaso-occlusive crisis and should be reported to the provider immediately. Therefore, the nurse should see this child first.

A nurse is providing teaching to the parent of an infant who has diaper dermatitis. The nurse should instruct the parent to apply which of the following to the affected area?
Zinc oxide

Diaper dermatitis is a common inflammatory skin disorder caused by contact with an irritant such as urine, feces, soap, or friction, and takes the form of scaling, blisters, or papules with erythema. Providing a protective barrier, such as zinc oxide, against the irritants allows the skin to heal.

A nurse is caring for a school-age child who is receiving chemotherapy and is severely immunocompromised. Which of the following actions should the nurse take?
Screen the child’s visitors for indications of infection.

A child who is severely immunocompromised is unable to adequately respond to infectious organisms, resulting in the potential for overwhelming infection. Therefore, the nurse should screen the child’s visitors for indications of infection.

A nurse is assessing a toddler who has gastroenteritis and is exhibiting manifestations of dehydration. Which of the following findings is the nurses priority?
Tachypnea

When using the airway, breathing, and circulation approach to client care, the nurse’s priority finding is the toddler’s tachypnea. Tachypnea is a result of the kidneys being unable to excrete hydrogen ions and produce bicarbonate, which leads to metabolic acidosis.

A nurse is teaching a school-age child who has a new diagnosis of type 1 diabetes mellitus. Which of the following statements by the child indicates an understanding of the teaching?
“I will give myself a shot of regular insulin 30 minutes before I eat breakfast.”

The child should administer regular insulin 30 min before meals so that the onset coincides with food intake.

A nurse is teaching the parent of an infant about ways to prevent sudden infant death syndrome (SIDS). Which of the following instructions should the nurse include?
“Give the infant a pacifier at bedtime.”

The nurse should inform the parent that protective factors against SIDS include breastfeeding and the use of a pacifier when the infant is sleeping.

The nurse is assessing a school-age child who has peritonitis. Which of the following findings should the nurse expect?
Abdominal distension

The nurse should identify that abdominal distention is an expected finding of peritonitis. Peritonitis is an inflammation of the lining of the abdominal wall. This inflammation in the abdomen, along with the ileus that develops, causes abdominal distention. Other manifestations include chills, irritability, and restlessness.

The nurse is interviewing the parent of an 18-month-old toddler during a well-child visit. The nurse should identify that which of the following findings indicates a need to assess the toddler for hearing loss?
The toddler received tobramycin during a hospitalization 2 weeks ago.

The nurse should identify tobramycin as an aminoglycoside, which is an ototoxic medication that can cause mild to moderate hearing loss, and should assess the toddler for a hearing impairment.

A nurse is providing teaching to the parent of a school-age child who has a new prescription for oral nystatin for the treatment of oral candidiasis. Which of the following instructions should the nurse include?
“Shake the medication prior to administration.”

The nurse should instruct the parent to shake the medication prior to administration to disperse the medication evenly within the suspension.

A nurse is admitting a school-age child who has Pertussis. Which of the following actions should the nurse take?
Initiate droplet precautions for the child.

The nurse should initiate droplet precautions for a child who has pertussis, also known as whooping cough. Pertussis is transmitted through contact with infected large-droplet nuclei that are suspended in the air when the child coughs, sneezes, or talks.

A nurse is caring for a toddler who is experiencing acute diarrhea and has moderate dehydration. Which of the following nutritional items should the nurse offer to the toddler?
Oral rehydration solution

A toddler who has acute diarrhea should consume an oral rehydration solution to replace electrolytes and water by promoting the reabsorption of water and sodium. This promotes recovery from dehydration.

A nurse in an emergency department is caring for a school-age child who is experiencing an anaphylactic reaction. Which of the following is the priority action by the nurse?
Administer epinephrine IM to the child.

When using the urgent vs. nonurgent approach to client care, the nurse should determine that the priority action is administering epinephrine IM to the child. During an anaphylactic reaction, histamine release causes bronchoconstriction and vasodilation. This is an emergency because ultimately this causes decreased blood return to the heart.

A nurse is planning care for a newly admitted schole-age child who has generalized seizure disorder. Which of the following interventions should the nurse plan to include?
Ensure the oxygen source is functioning in the childs room: The nurse should recognize that maintaining the child’s airway is important during a seizure. The nurse should ensure that the oxygen source is functioning because the child might require supplemental oxygen following a seizure.

A nurse is providing dietary teaching to the guardian of a school-age child who has cystic fibrosis. Which of the following statements should the nurse make?
“You should offer your child high-protein meals and snacks throughout the day.” The nurse should instruct the guardian to provide a diet that is well-balanced and high in protein and calories. Children who have cystic fibrosis require a higher percentage of the recommended dietary allowances of all nutrients to meet their energy requirements. Children who have good nutritional intake have improved lung function and decreased risk of infection.

A nurse is providing discharge teaching to the parents of a 6-month-old infant who is postoperative following hypospadias repair with a stent placement. Which of the following instructions should the nurse include in the teaching?
“Allow the stent to drain into your infants diaper.” The nurse should instruct the parents to ensure that the stent drains directly into the infant’s diaper to prevent kinking or twisting that can interfere with urine flow.

A nurse is caring for a school-age child who has primary nephrotic syndrome and is taking prednisone. Following 1 week of treatment, which of the following manifestations indicates to the nurse that the medication is effective?
Decreased edema: A child who has nephrotic syndrome can experience edema due to the increased glomerular permeability, which increases protein loss. Prednisone decreases glomerular permeability, which causes fluid to shift from the extracellular spaces, resulting in decreased edema.

A nurse is receiving change-of-shift report for four children. Which of the following children should the nurse assess first?
A toddler who has a concussion and an episode of forceful vomiting.: When using the urgent vs. nonurgent approach to client care, the nurse should assess this child first. An episode of forceful vomiting is an indication of increased intracranial pressure in a toddler who has a concussion.

A nurse is providing discharge teaching to the guardians of a toddler who had lower leg cast applied 24 hr ago. The nurse should instruct the guardians to report which of the following finding to the provider?
Restricted ability to move the toes.: The nurse should inform the guardians that a restricted ability of the toddler to move their toes is an indication of neurovascular compromise and requires immediate notification of the provider. Permanent muscle and tissue damage can occur in just a few hours.

A nurse in an emergency department is auscultating the lungs of an adolescent who is experiencing dyspnea. The nurse should identify the sound as which of the following?
Wheezes: The nurse should identify the sound during auscultation as wheezes, which are high-pitched, musical or whistling-like sounds heard primarily on expiration as air passes through and vibrates narrowed airways.

A nurse is caring for a preschooler who has congestive heart failure. The nurse observes wide QRS complexes and peaked T waves on the cardiac monitor. Which of the following prescriptions should the nurse clarify with the provider?
Potassium Chloride: The nurse should identify that a child who has congestive heart failure can develop electrolyte imbalances, such as hyperkalemia or hypokalemia. The nurse should identify that the child is exhibiting manifestations of hyperkalemia and contact the provider about the administration of potassium chloride, which can increase the severity of hyperkalemia.

A nurse is planning an educational program for school-age children and their parents about bicycle safety. Which of the following information should the nurse plan to include?
The child should be able to stand on the balls of their feet when sitting on the bike.: To decrease the risk for injury, parents should ensure that the bike is the correct size for the child. When seated on the bike, the child should be able to stand with the ball of each foot touching the ground and should be able to stand with each foot flat on the ground when straddling the bike’s center bar.

A nurse is monitoring the oxygen saturation level of an infant using pulse oximetry. The nurse should secure the sensor to which of the following areas on the infant?
Great Toe. The nurse should secure the sensor to the great toe of the infant and then place a snug-fitting sock on the foot to hold the sensor in place. The nurse should also check the skin under the sensor site frequently for temperature, color, and the presence of a pulse.

A nurse is an emergency department is caring for a school-age child who has epiglottitis. Which of the following actions should the nurse take?
Monitor the childs oxygen saturation: The nurse should monitor the child’s oxygen saturation level because the child is experiencing acute respiratory distress and it is necessary to determine if the child is responding to treatment.

A nurse in an emergency department is caring for a school-age child who has sustained a minor superficial burn from fireworks on their forearm. Which of the following actions should the nurse take?
Apply an antimicrobial ointment to the affected area.: The nurse should apply an antimicrobial ointment to the burned area to prevent infection.

A nurse in a providers office is caring for a school-age child who has varicella. The parents asks the nurse when their child will no longer be contagious. Which of the following responses should the nurse make?
“When your childs lesions are crusted, usually 6 days after they appear.”: The nurse should inform the parent that the child is contagious 1 day prior to lesion eruption and until the vesicles have crusted over, which usually takes about 6 days.

A nurse is providing discharge teaching to the parent of a school-age child who has moderate persistant asthma. Which of the following instructions should the nurse include?
“Pulmonary function tests will be performed every 12 to 24 months to evaluate how your child is responding to therapy.”: The nurse should inform the parent that their child will need pulmonary function tests every 12 to 24 months to evaluate the presence of lung disease and how the child is responding to the current treatment regimen. As children grow, sometimes their manifestations can improve or decline, and treatment needs to change accordingly.

A nurse is admitting an infant who has intussusception. Which of the following findings should the nurse expect? (Select all that apply.)
-Vomiting
-Lethargy Vomiting is correct. The nurse should expect an infant who has intussusception to exhibit vomiting due to the obstruction that occurs when a segment of the bowel telescopes within another segment of the bowel.Lethargy is correct. The nurse should expect an infant who has intussusception to exhibit lethargy due to episodes of severe pain during which the infant cries inconsolably, leading to exhaustion and decreased nutritional intake.

A nurse is reviewing the laboratory results of a school-age child who is 1 week postoperative following an open fracture repair. Which of the following findings should the nurse identify as an indication of a potential complication?
Erythrocyte sedimentation rate 18 mm/hr: The nurse should identify that an erythrocyte sedimentation rate of 18 mm/hr is above the expected reference range of up to 10 mm/hr and is an indication of osteomyelitis.

A nurse is providing discharge teaching to the parents of a 3-month old infant following a cheiloplasty. Which of the following instructions should the nurse include?
“Apply a thin layer of antibiotic ointment on the your babys suture line daily for the next 3 days.”: The nurse should instruct the parents to apply a thin layer of antibiotic ointment on the infant’s suture line daily for 3 days and then continue to apply petroleum jelly to the area for several weeks to promote healing.

A nurse is discussion organ donation with the parents of a school-age child who has sustained brain death due to a bicycle crash. Which of the following actions should the nurse take first?
Explore the parents feelings and wishes regarding organ donation.: The first action the nurse should take when using the nursing process is assessment. The nurse should first explore the parents’ feelings and wishes regarding organ donation to assist in determining if organ donation is the right choice for the family.

A nurse is caring for a 1-month-old infant who is breastfeeding and requires a heel stick. Which of the following actions should the nurse take to minimize the infants pain?
Allow the mother to breastfeed while the sample is being obtained.: The nurse should allow the mother to breastfeed the infant prior to or during the procedure. Evidence-based practice indicates breastfeeding or non-nutritive sucking with a pacifier can provide nonpharmacological pain management in infants.

A nurse is assessing an adolescent who received a sodium polystyrene sulfonate enema. Which of the following findings indicates effectiveness of the medication?
Serum potassium level 4.1 mEq/L, The nurse should monitor the adolescent’s serum potassium level following the administration of sodium polystyrene sulfonate. This medication is used to treat hyperkalemia by exchanging sodium ions for potassium ions in the intestine. Therefore, a potassium level within the expected reference range of 3.4 to 4.7 mEq/L indicates the effectiveness of the medication.

A charge nurse is preparing to make a room assignment for a newly admitted school-age child. Which of the following considerations is the nurses priority?
Disease process: The transmission of infectious diseases is the greatest risk to this child and other children on the unit. Therefore, the child’s disease process is the nurse’s priority consideration

A nurse is assessing the pain level of a 3-year-old toddler. Which of the following pain assessment scales should the nurse use?
FACES: The nurse should use the FACES pain rating scale for pediatric clients who are 3 years old and older. This scale allows the toddler to point to the face that depicts their current level of pain. The nurse can then determine the need for pain management.

A nurse is preparing to administer ibuprofen 5 mg/kg every 6 hr PRN for a temperatures above 38.0 C (100.5 F) to an infant who weighs 17.6 lb. Available is ibuprofen oral suspension 100mg/5mL. How many mL should the nurse administer to the infant per dose?
2 mL

A nurse is assessing a 6-month-old infant during a well-child visit. Which of the following findings should the nurse report to the provider?
Presence of strabismus: Strabismus, or crossing of the eyes, typically disappears at 3 to 4 months of age. If not corrected early, this can lead to blindness. Therefore, the nurse should report this finding to the provider.

A school nurse is caring for a child following tonic-clonic seizure. Which of the following actions should the nurse take first?
Check the childs respiratory rate.: When using the airway, breathing, and circulation approach to client care, the nurse should determine the priority action is to assess the child’s respiratory rate. If the child is not breathing, the nurse should administer rescue breaths.

A nurse is planning developmental activities for a newly admitted 10-year-old child who has neutropenia. Which of the following actions should the nurse plan to take?
Provide the child with a book about adventure.: The nurse should provide a school-age child with a book about adventure as a developmental activity because children are expanding their knowledge and imagination during this age. Through reading, school-age children can feel powerful and skillful as they imagine themselves in the stories they read.

A nurse in a health department is caring for an emancipated adolescent who has an STI and is unaccompanied by a guardian. Which of the following actions should the nurse take?
Have the adolescent sign a consent form for treatment.: The nurse should identify that an emancipated minor can sign the consent form for treatment of an STI or any other form of medical treatment requiring consent.

A nurse is assessing an 8-year-old child who has early indications of shock. After establishing an airway and stabilizing the childs respirations, which of the following actions should the nurse take next?
Initiate IV access.: After establishing an airway and stabilizing the child’s respirations, the next action the nurse should take when using the airway, breathing, and circulation approach to client care is to establish IV access to maintain the child’s circulatory volume.

A nurse is performing hearing screenings for children at a community health fair. Which of the following children should the nurse refer to a provider for a more extensive hearing evaluation?
An 8-month-old who is not yet making babbling sounds.: The nurse should refer an infant who is not making babbling sounds by the age of 7 months to a provider for a more extensive evaluation of hearing

A nurse is providing discharge teaching to the guardian of a school-age child who has undergone a tonsillectomy. Which of the following statements by the guardian indicates an understanding the teaching?
“I will notify the doctor if I notice that my child is swallowing frequently.”

A community health nurse is assessing an 18-month-old toddler in a community day care. Which of the following findings should the nurse identify as a potential indication of physical neglect?
Poor personal hygiene: A toddler who has poor personal hygiene can be a potential indication of physical neglect. Because toddlers are still dependent on their parents or guardians for help with hygiene needs, poor personal hygiene can indicate a lack of supervision.

A nurse assessing a school-age child who has an infratentorial brain tumor. Which of the following findings should the nurse identify as a manifestation of increased intracranial pressure?
Difficulty concentrating: The nurse should identify that irritability, inability to follow commands, and difficulty concentrating are manifestations of increased intracranial pressure due to decreased blood flow within the brain and pressure on the brainstem.

A nurse is providing teaching to an adolescent about how to manage tinea pedis. Which of the following statements by the adolescent indicates an understanding of the teaching?
“I should wear sandals as much as possible.”: Sandals allow air to circulate around the feet, decreasing perspiration and eliminating the medium for bacteria and fungus to grow. The nurse should inform the adolescent that wearing sandals, open-toed, or well-ventilated shoes will promote healing of the fungal infection.

A nurse is caring for a school-age child who has diabetes mellitus and was admitted with a diagnosis of diabetic ketoacidosis. When performing the respiratory assessment, which of the following findings should the nurse expect?
Deep respirations of 32/min: The nurse should expect Kussmaul respirations in a child who has diabetic ketoacidosis. These deep and rapid respirations are the body’s attempt to eliminate excess carbon dioxide and achieve a state of homeostasis.

A nurse is planning n educational program to teach parents about protecting their children from sunburns. Which of the following instructions should the nurse plan to include?
“Choose a waterproof sunscreen with a minimum SPF of 15.”: The nurse should instruct parents to apply a waterproof sunscreen with a minimum SPF of 15 for children. The parents should apply the sunscreen prior to sun exposure to reduce the risk of sunburn.

A nurse is providing teaching to the parents of a preschooler who has heart failure and a new prescription for digoxin twice daily. Which of the following instructions should the nurse include in the teaching?
“Brush the childs teeth after giving the medication.”: The nurse should teach the parent to closely monitor the child’s number of wet diapers. Monitoring the number of wet diapers per day is an effective way for the parent to monitor adequate output and hydration status.

A nurse is providing teaching to the family of a school-age child who has juvenile idiopathic arthritis. Which if the following instructions should the nurse include in the teaching?
“Encourage the child to perform independent self-care.”: The nurse should teach the family the importance of encouraging the child to perform independent self-care. This will minimize the child’s pain while maximizing mobility. Encouraging and praising the child’s efforts for independence will also increase their self-esteem.

A nurse is creating a plan of care for a child who has varicella. Which of the following interventions should the nurse include?
Initiate airborne precautions for the child.: The nurse should initiate airborne precautions for a child who has varicella because it is spread through droplets in the air. The incubation period for varicella is 2 to 3 weeks, and the child is contagious even before lesions appear.

A nurse is assessing a school-age child who has an acute spinal cord injury following a sports injury 1 week ago. Identify the area the nurse should tap to elicit the bicep reflect.
Correct answer is A

A school nurse is providing an in-service for faculty about improving education for students who have ADHD. Which of the following statements by a faculty member indicates an understanding of the teaching?
“I will teach challenging academic subjects to students who have ADHD in the morning.” Faculty should plan to teach challenging academic subjects in the morning when students who have ADHD are most able to focus and their medication is most likely to be effective

A nurse is caring for a school-age child who has peripheral edema. The nurse should identify that which of the following assessments should be performed to confirm peripheral edema?
Palpate the dorsum of the childs feet: The nurse should palpate the dorsum of the feet by pressing the fingertip against a bony prominence for 5 seconds to assess for peripheral edema.

A nurse is caring for an infant who is receiving IV fluids for the treatment of Tetralogy of Fallot and begins to have hypercryanotic spell. Which of the following actions should the nurse take?
Place the infant in a knee-chest position: The nurse should place the infant in a knee-chest position during a hypercyanotic spell to decrease the return of desaturated venous blood from the legs and to direct more blood into the pulmonary artery by increasing systemic vascular resistance.

A nurse is reviewing the dietary choices of an adolescent who has iron deficiency anemia. The nurse should identify that which of the following menu items has the highest amount of nonheme iron?
½ cup raisins: The nurse should encourage the adolescent to eat raisins because they contain the highest amount of nonheme iron.

A nurse in an emergency department is assessing a 3-month-old infant who has rotavirus and is experiencing acute vomiting and diarrhea. Which of the following manifestations should the nurse identify as an indication that the infant has moderate to severe dehydration?
Sunken anterior fontanel: The nurse should recognize that a sunken anterior fontanel is an indication of moderate to severe dehydration due to the acute loss of fluid.

A nurse is planning care for a school-age child who has tunneled central venous access device. Which of the following interventions should the nurse include in the plan?
Use a semipermeable transparent dressing to cover the site: The nurse should cover the site with a semipermeable transparent dressing to reduce the risk of infection.

A nurse is teaching a group of parents about infectious mononucleosis. Which of the following statements by parent indicates an understanding the teaching?
“Mononucleosis is caused by an infection with the Epstein-Barr virus.”: The nurse should identify that mononucleosis is a mildly contagious illness that occurs sporadically or in groups, and is primarily caused by the Epstein-Barr virus.

A nurse is caring for a newly admitted school-age child who has hypopituitarism. Which of the following medications should the nurse expect the provider to prescribe?
Recombinant growth hormone: Recombinant growth hormone injections are used to treat hypopituitarism, which inhibits cell growth and results in growth failure. The nurse should expect the provider to prescribe this treatment.

A nurse is creating a plan of care for a preschooler who has Wilms’ tumor and is scheduled for surgery. Which of the following interventions should the nurse include?
Avoid palpating the abdomen when bathing the child before surgery.: The nurse should avoid palpating the abdomen when bathing the child before surgery because movement of the tumor can cause cancer cells to disseminate to other sites, adjacent and distant to the tumor site.

A nurse is providing discharge teaching to the parent of an 18-month-old toddler who has dehydration due to acute diarrhea. Which of the following statements by the parent indicates an understanding of the teaching?
“I will monitor my childs number of wet diapers.”: The nurse should teach the parent to closely monitor the child’s number of wet diapers. Monitoring the number of wet diapers per day is an effective way for the parent to monitor adequate output and hydration status.

A nurse is teaching the guardian of a 6-month-old infant about teething. Which of the following statements should the nurse make?
“Your baby might pull at their ears when they are teething.”

A nurse is creating a plan of care for a newly admitted adolescent who has bacterial meningitis. How long should the nurse plan to maintain the adolescent in droplet precautions?
For 24 hr following initiation of antimicrobial therapy: The nurse should plan to maintain the adolescent on droplet precautions for at least 24 hr following initiation of antimicrobial therapy. This practice will ensure that the adolescent is no longer contagious, which protects family members and the personnel caring for the client. Prophylactic antibiotics might be prescribed to individuals who were in close contact with the adolescent.

A nurse is providing anticipatory guidance to the parent of a toddler. Which of the following expected behavior characteristics of toddlers should the nurse include?
Expressed likes and dislikes: The nurse should include that expressing likes and dislikes is an expected behavior of toddlers. This is the time in life when a toddler is developing autonomy and self-concept. They will try to assert themselves and frequently refuse to comply. The parent should allow the child to have some control, but also set limits for them so they learn from their behavior and learn to control their actions.

A nurse is admitting a 4-month-old infant who has heart failure. Which of the following findings is the nurses priority?
Episodes of vomiting: When using the urgent vs. nonurgent approach to client care, the nurse should determine that the priority finding is three episodes of vomiting. This can indicate digoxin toxicity, which requires immediate intervention. Therefore, this is the nurse’s priority finding.

A nurse in an emergency department is assessing a toddler who has Kawasaki disease. Which of the following findings should the nurse expect? (Select all that apply.)
-Increased temperature
-Xerophthalmia
-Cervical lymphadenopathy Increased temperature is correct. Kawasaki disease is an acute illness associated with a fever that is unresponsive to antipyretics or antibiotics.Gingival hyperplasia is incorrect. Children who have Kawasaki disease develop a strawberry tongue, cracked lips, and edema of the oral mucosa and pharynx. A child who is receiving phenytoin therapy can develop gingival hyperplasia.Xerophthalmia is correct. Ophthalmic manifestations of Kawasaki disease include reddening of the conjunctiva and dryness of the eyes, or xerophthalmia.Bradycardia is incorrect. Kawasaki disease is an infection that affects the vascular system, including the heart. The nurse should expect the child to be tachycardic with a gallop rhythm. Long-term effects of Kawasaki disease include the development of coronary artery aneurysms or myocardial infarction.Cervical lymphadenopathy is correct. A child who has Kawasaki disease can develop enlarged cervical nodes on one side of the neck that are nontender and greater than 1.5 cm in size.

A nurse is caring for a 10-year-old child following a head injury. Which of the following findings should the nurse identify as an indication that the child is developing diabetes insipidus?
Sodium 155 mEq/L: A child who has a head injury can develop diabetes insipidus as a result of pituitary hypofunction leading to a deficiency of antidiuretic hormone. Underexcretion of antidiuretic hormone leads to polyuria and polydipsia, and possibly dehydration. With the excessive loss of free water, sodium levels rise above the expected reference range of 136 to 145 mEq/L.

A nurse is planning care to address nutritional needs for a preschooler who has cystic fibrosis. Which of the following interventions should the nurse include in the plan?
Increase fat content in the childs diet to 40% of total calories.: A child who has cystic fibrosis is unable to properly digest fats due to fibrosis of the pancreas and limited secretion of pancreatic enzymes. The nurse should increase the child’s fat intake to 35% to 40% of total caloric intake.

A nurse is caring for a toddler who has acute otitis media and a temperature of 40 C (104 F). After administering acetaminophen, which of the following actions should the nurse plan to take to reduce the toddler’s temperature?
Dress the toddler in minimal clothing.: The nurse should recognize that dressing the toddler in minimal clothing will expose the skin to air and maximize heat evaporation from the skin, thus reducing the toddler’s temperature.

A nurse is teaching a school-age child and their parent about postoperative care following cardiac catheterization. Which of the following instructions should the nurse include?
“Wait 3 days before taking a tub bath.”: The child should keep the site clean and dry for at least 3 days to reduce the risk of infection. Tub baths should be avoided for 3 days to avoid immersion of the incision in water.

A nurse is assessing an infant who has pneumonia. Which of the following findings is the priority for the nurse to report to the provider?
Nasal flaring: When using the airway, breathing, and circulation approach to client care, the nurse should determine that the priority finding to report to the provider is nasal flaring. Nasal flaring indicates the infant is experiencing acute respiratory distress.

  1. A nurse is teaching the parents of a toddler who has cognitive impairment about toilet training. Which of the following instructions should the nurse include in the teaching?
    “Award your child with a sticker when they sit on the potty chair.”: A child who has a cognitive impairment learns through shaping behaviors. The parents should reward the child for sitting on the potty chair as a reinforcement of the desired behavior of continence. As the child repeats this action, the parents can gradually decrease this reward and then give rewards for the next step in the task, such as voiding while sitting on the potty chair.

EDKH
Epstein Didn’t Kill Himself

(ADHD) A nurse is providing teaching to the parents of a school aged child who has ADHD. Which of the following instructions should the nurse include?
Place the childs daily activities on an organizational chart

(Sickle cell) A nurse is caring for a child who has sickle cell anemia. Which of the following findings is
a priority for the nurse to report to the provider?
Facial twitching

(Tonsillectomy) A nurse in the PACU is caring for a school aged child immediately after a tonsillectomy. Which of the following actions should the nurse take?
Place the child in a side lying position

(STIs) A nurse in the ER is caring for an adolescent who was requesting testing for STIs. Which
of the following actions is appropriate for the nurse to take?
Obtain the written consent from the client

(Ferrous sulfate) A nurse is providing teaching about medication administration to the parents of the toddler who has a new prescription for liquid ferrous sulfate. Which of the following instructions should the nurse include?
Dilute the drops of water prior to administration

(Phenylketonuria) A nurse is providing dietary teaching to a parent of a 10-month-old infant who has
phenylketonuria. Which of the following responses by the parent indicates an understanding of the teaching?
I will steam carrots and will cut them into small pieces for her

(Appendectomy) a nurse is caring for a child who s 2 days post-op following an appendectomy due to the
rupture of the appendix the childs NG tube is set to low intermittent suction. Which of the following indicates that the childs GI function has returned?
. The nurse auscultates bowel sounds

(Diaper rash) A nurse is providing teaching to the parent of an infant who has a diaper rash. Which of
the following statements by the parents indicate an understanding of the teaching?
I will use superabsorbent disposable diapers

(Sickle cell crisis) a nurse is administering an opioid to an adolescent who is in a sickle cell crisis. Which statement is true regarding opioid pain management?
Oral opioid does should be larger than parenteral doses

(Shunt displaced) A nurse is providing discharge to the parents of a school aged child following placement
of a ventriculoperitoneal shunt. The nurse should determine the teaching was effective when the parents identify which of the following as an indicator that the shunt has been displaced?
Elevated temperature

(Labs) A nurse is reviewing laboratory results of a school aged child. Which of the following
findings to the nurse reports the provider?
Platelets 110,000

(Chest tube) A nurse is planning care for a child immediately following the insertion of a chest tube for continuous suction with a closed drainage system. Which of the following interventions should the nurse include in the plan of care?
Ensure continuous bubbling is present in the suction control chamber

(Digoxin) A nurses caring for an infant who has heart failure and is receiving digoxin. Which of the following findings indicates a positive response to the medication?
Urine output 2 mL/kg/hr

(RSV) A nurse is assessing a six month old infant who has respiratory syncytial virus. The nurse
should immediately report which of the following findings to the provider?
Tachypnea

(Newborn screen) A nurse is reviewing the results of the newborn screening of a newborn who is one week old. Results include total T4 0.8mcg/dl phenylalanine 0.7 and negative galactosemia. Which of the following nursing include in the plan of care?
nstruct the newborns parent about how to administer levothyroxine

(Diarrhea) A nurse is caring for a newly admitted toddler who has acute diarrhea. Which of the
following actions should the nurse take first.
Initiate contact precautions

(Cleft palate repair) A nurse is caring for toddler who post operative following cleft palate repair. Which of the following actions should the nurse take?
Administer opioids for mouth pain

(Measles) A nurse is assessing a child who has measles. Which of the following areas should the
nurse inspect for koplik spots?
Inside the mouth

(Otitis media) A nurse is assessing an infant who has acute otitis media which of the following findings should the nurse expect? Select all that apply.
a. Crying
b. Fever
c. Restlessness

(Gas inhalation) A nurse in the emergency department is assessing an adolescent who reports inhalation
of gasoline. Which of the fallowing should the nurse expect?
Ataxia

(Dehydration) A nurse caring for a 4 year old child who has moderate dehydration. Which of the fallowing should the nurse expect?
Orthostatic hypertension

(Lactose intolerance) A nurse is planning to teach an adolescent who is lactose intolerant about dietary
guidelines. Which of the fallowing instructions should the nurse include in the teaching?
You can replace milk with non dairy source of calcium

(Skeletal traction) A nurse is caring for a school age child who is in 90/90 skeletal traction. Which of the fallowing actions should the nurse take?
Place the child on an alternating pressure mattress

(Radiation therapy) A nurse is providing teaching to the parents of the child who is receiving radiation
therapy. Which of the fallowing instructions should the nurse include in the teaching? (select all that apply)
-Avoid giving your child lengthy baths
-Encourage mild activity daily
-Dress your child in loose fitting clothing

(Diabetes) A nurse is assessing a school age child who has type 1 diabetes mellitus. The nurse notes that the child is diaphoretic. Which of the following actions should the nurse take?
Obtain blood glucose level

(CF) A nurse is providing teaching to a parent of a child who has cystic fibrosis and a new
prescription for dornase alfa. Which of the following instructions should the nurse include in the teaching?
store the medication in the refrigerator

(Gentamicin) A nurse is caring for a child who is to receive first dose IV gentamycin which of the
following actions should the nurse take?
Maintain strict I/O

(Immunizations) A nurse is preparing to administer immunizations to a 5 year old child who is up to date with the current immunization schedule. Which of the following immunizations should the nurse plan to administer?
Varicella

(CF testing) A nurse is providing teaching to the parents of an infant who is to under go pilocarpine
iontophoresis testing for cystic fibrosis. Which of the following statements should the nurse include in the teaching?
Test will measure the amount of chloride in the babys sweat

(Terminal illness) A nurse is caring for a child who is terminally ill. The parents tell the nurse that the child
will be fine because they know another child who survived the same illness. Which of the following responses should the nurse make?
Tell me what you know about your childs illness

(Impetigo)A nurse is providing teaching to the parents of a child who has impetigo. Which of the
following instructions should the nurse include in the teaching?
Apply bacterial ointment to lesions

(Meningitis) A nurse is caring for a child who has bacterial meningitis, which of the following criteria should indicates the nurse should remove the child from droplet precautions?
antibiotics initiated 24 hour ago

(Assessment) A nurse is prioritizing care for four clients which of the following clients should the nurse
assess first?
An adolescent who has sickle cell anemia and slurred speech

(CF) A nurse is assessing a toddler who has cystic fibrosis. Which of the following findings should the nurse expect?
Steatorrhea

(Death response) nurses in a family practice clinic is assessing a preschool aged child who recently
experienced the death of a sibling. Which of the following reactions is an age appropriate response to death?
The child eels responsible to the childs death

(Vitals) A nurse in a providing office is assessing the vital signs of a 1 year old toddler which of
the following findings should the nursed report to the provider?
Respiratory rate 54

(Visual acuity) A nurse is preparing to assess a 4 year old child visual acuity. Which of the following actions should the nurse prepare to take?
Test the child without glasses before testing with

(HIv) A nurse is providing teaching to a parent of a child who has HIV. Which of the following
statement by the parent indicates an understanding of the teaching?
My child will need to repeat the childhood immunization once hes in remission

(Terminal illness) A nurse is caring for a preschool-age child who has a terminal illness. Which of the following should the nurse…
Believes the condition is permanent

(Sickle cell) A nurse is planning care for a child who was experiencing sickle cell crisis. Which of the
following interventions should the nurse have in plan of care?
Administer meperidine as needed for pain

(Arm cast) A nurse educating an adolescent following an application of an arm cast. Which of the following statements by the client indicates an understanding of the teaching?
I should limit the use of my fingers of my broken arm

(Immunizations) A nurse is providing teaching to a 2 month old infant about immunization schedules
which of the following statements from the parents indicate an understanding of the teaching?
My child needs to get the MMR immunization when she is 12 months old

(Safety) A nurse in providing injury prevention to the parents of the toddler which of the
following safety measures should the nurse incorporate in the teaching.
Check clothing for loose buttons

(ICp) indicates that the infant is experiencing ICP?
Irritability

(Assessment)A nurse in a urgent care clinic is prioritizing care for 4 children which of the children should the nurse asses first?
A toddler who has nephrotic syndrome and facial edema

(Asthma)a nurse in a emergency department is assessing a school age child who has asthma of
the following findings should the nurse prioritize?
Decrease breath sounds

(Diarrhea & dehydration) A nurse is providing to a parent to an 11 month old infant who has acute diarrhea ad dehydration what should the nurse instruct the parent to provide to the infant
oral electrolyte solution

(Immunization) A nurse is preparing to administer an immunization to a 3 month old infant which of the
following is relives atraumatic care?
Provide a pacifier coated with oral sucrose solution prior to the injections

(Osteomyelitis)A nurse is admitting a school age who has osteomyelitis which of the following actions…..
Obtain a blood culture

(Assessment) a nurse is assessing a 24 month toddler which of the finding should….?
Has a vocabulary of 40 words

(Seizures) a nurse is providing teaching to the parents of a school age child newly diagnosed with a
seizure disorder. The nurse should teach the parents which of the following actions during a seizure?
Clear the area of hard objects

(Immunization) A nurse is providing anticipatory guidance to a parents of a 1 month old infant. The
nurse should include that it is recommended to start the series of which of the following immunization first?
Inactivated poliovirus

(Glomerulonephritis) A nurse is providing education about dietary modifications to the parents of the school
age child who has glomerulonephritis. Which of the following information should the nurse include in the teaching?
Decrease the child sodium intake

(Diabetes) A nurse is assessing an adolescent who has type 1 diabetes mellitus which of the
fallowing findings is the nurse priority?
Glycosuria

(Hemolytic uremic shndrome) A nurse is reviewing the laboratory report of a toddler was has hemolytic uremic syndrome. Which lab values should the nurse expect?
BUN of 28

(Sickle cell) A nurse is caring for a school age child who is experiencing a sickle cell crisis. Which of
the following actions should the nurse take?
Apply warm compress to the infected joints

(Open heart surgery) A nurse on a pediatric intensive care unit is caring for a toddler who weighs 12kg and is postoperative following an open heart surgery. Which of the following findings should the nurse report to the provider?
Posterior and tibial pulse 2+

(Celiac disease) A nurse is providing teaching to the parents of a preschool age child who has celiac
disease. Which of the following instructions should the nurse include?
Your child will be on a gluten free diet for the rest of her life

(Tonsillectomy) A nurse is caring for a school age child who is 1 hour post op following a tonsillectomy. Which of the following actions should the nurse take
-Admin analgesics to the child on a schedule basis
-Observe the child for frequent swallowing
-And discourage coughing

(Asthma) A nurse admin albuterole by metered dose inhaler for a preschool aged child who is experiencing an asthma exacerbation. Which of the following should the nurse report to the provider?
Intercostal retractions

(Heart failure) A nurse is caring for a school age child who has heart failure. Which of the following
finding should the nurse expect?
-Tachycardia
-Cyanosis
-Dyspnea
-Bounding peripheral pulses

(Head injury) A nurse in the emergency department is assessing a toddler who has a head injury.
Which of the following findings should the nurse report to the provider?
Vomiting

(Neuroblastoma) A nurse is caring for a toddler who is terminal stage of neuroblastomas, the parent ask how can we help our child know which of the following responses by the nurse is appropriate?
Stay close to your child

(Well baby visit) During a well baby visit the parents of a 2 week old new born tells the nurse my baby
always keeps her head tilted to the right side. The nurse should further assess which of the following areas?
Sternocleidomastoid muscle

(Well baby visit) A nurse is caring for a single mother of a 6 month old infant. During a well baby visit the
mother expresses feelings inexperienced in care of the baby. The nurse should recommend which of the following community resource.
Parent enhancement center

(GERD) A nurse is admitting an infant who has GERD which of the following is priority
assessment finding?
Excessive crying

(Dehydration) A nurse is caring for an infant that has severe dehydration. Which clinical finding should the nurse expect?
Rapid respirations

(Healthy eating) A nurse is teaching a group of female adolescent about healthy eating. Which of the
following instructions should the nurse include in the teaching?
Consume 1500 to 1700 calories per day

(Cf) a nurse is providing discharge teaching to the parents of a school age child who has cystic fibrosis which response by the parents indicates an understanding if the teaching?
I will give my child pancreatic enzyme with snacks and meals

(Impetigo) A nurse is caring for a child who has impetigo contagiosa that developed in the hospital.
Which of the following actions should the nurse take?
Initiate contact isolation precaution

(Meningitis) A nurse is caring for a 4 year old child who has meningitis and is receiving gentamacin. Which of the following labs values should the nurse to the provider
Creatinine 1.4mg

(Adhd) A nurse is providing teaching to the parents of a school age child who has ADHD and a
new prescription of for methylphenidate. The nurse should explain that this medication should have which of the following therapeutic effects?
Increasing focus

(Cf) A nurse is teaching an adolescent how to manage his cystic fibrosis. Which of the
following statements by the adolescent indicates understanding of the teaching?
I will increase my intake of vitamin D

(Vein puncture) A nurse is preparing to perform a venipuncture to collect a blood sample from an infant which of the following restraints should the nurse plan to use for this procedure?
Mummy

(Epiglottis)A nurse in the providers office is caring for a preschool age child who might have a acute
epiglottitis which of the following actions should the nurse take?
provide humidifier oxygen via nasal cannula

(Rheumatic fever) A nurse is reviewing the laboratory report of A school age Child who has rheumatic fever which of the following laboratory findings should the nurse expect?
Increased antistreptolysin O titer

(Diverticulum)A nurse is planning care for adolescent follow repair of Meckel diverticulum which of the
following actions should the nurse include in the plan of care?
Maintain a NG tube for decompression

(Peritoneal dialysis) A nurses preparing to perform peritoneal dialysis for your child was elevated serum creatinine level after explaining the procedure which of the following action should the nurse plan to take?
Obtain child’s weight

(Appendectomy)A nurses caring for an adolescent who is one hour postoperative following
appendectomy which of the following findings should the nurse report to the provider?
Muscle rigidity

(Tonsillectomy) And nurses caring for preschool age child whose postoperative following a tonsillectomy and is clearing her throat frequently which of the following action should take first?
Give the child small sips of water

(Chemotherapy ulcers) And nurses planning care for a toddler who has developed oral ulcers in response to
chemotherapy which of the following actions should the nurse include in the plan of care?
Cleanse the gums with saline soaked gauze

(Chest tube insertion) In nurses planning care for the child immediately following her insertion of a chest tube
for continuous suctioned with a close drainage system which of the following intervention should the nurse include in the plan of care?
ensure continuous bubbling his present in the suction control chamber

(Cushings)a nurse is assessing an adolescent who has Cushing syndrome which of the following
findings should the nurse expect
blood glucose of 320

(FTT) A charge nurse is planning care for an infant who has failure to thrive which of the following actions should the nurse include in the plan of care?
Use half strength formula when feeding the infant

(Cardiac cath) A nurse is providing postoperative care for a child following an arterial cardiac
catheterization which of the following actions should the nurse take?
Keep affected extremity straight for about 6 hours

(SIDs) A nurses providing discharge teaching to the parents of an infant who is at risk for sudden infant death syndrome which of the following statements by the parents indicate an understanding of the teaching
I will dress my baby in light weight clothing to sleep

(Down’s syndrome) A nurse in the providers office is providing teaching to the parents of a preschooler who
has down syndrome which of the following statements buy one of the parents indicates an understanding of instructions?
We’ll be sure to demonstrate a new skill before expecting our Son to perform it.

(Safety) A nurse is teaching a parent of a 10 month old about home safety what should be
included in the teaching
-Place gates on top and bottom of stairs
-Ensure that the crib mattress is in the lowest position

(Diarrhea) A nurse is assessing a 3 month old infant who has diarrhea which of the following should
the nurse expect?
Increased hemocratic

(Umbilical hernia) A nurse is providing teaching to a parent of an infant who has 1 cm umbilical hernia which of the following instructions should the nurse include in the teaching
the bulge can temporarily enlarge when your baby cries

(Pertussis) A nurse is admitting for pertussis which transmission based precaution should the nurse
initiate
Droplet percussion

(Lead poisoning) In Nurse is assessing a toddler who has a history of lead poisoning which of the following actions should the nurse take?
Perform developmental testing for delays

(Physical abuse) A nurse is assessing a seven -year-old student the nurse should identify which of the
following finding as a potential indicator for physical abuse
Bruising around the wrist

(Leukemia) In nurses reviewing the medical record of a 24 month old child who has acute lymphocytic leukemia which of the following actions should the nurse take
Initiate bleeding precautions

(Iron def anemia) A nurses providing discharge teaching to the parents of a father who was iron deficiency
anemia and a new prescription for Ferrous sulfate which of the following instructions for the nurse include
Administer the medication to your child with the dropper

(Diabetes)Hey nurses providing teaching to the parent of the school’s child was diabetes mellitus
about managing diabetes during illness which of the following statements by the parents and the kids understanding of teaching?
I’ll increase them on the fluids offer to my child

(Tetralogy of fallot)A nurse is caring for an infant who has tetralogy of fallot and is having a hypercyanotic
episode after crying which of the following intervention should the nurse implement?
placed infant in knee chest position

(Pediculosis) A nurse is planning an inservice for parents of school age children about the tx of pediculosis cpaitis, which of the following instructions for the nurse to plan including in the teaching?
remove nits from child’s hair using a fine tooth comb

(Acetaminophen) A nurse in the emergency room is caring for a child who is 18 kg and injustice six
500 mg acetaminophen about four hours ago which of the following actions and there’s take?
Prepare to give oral N-acetylcysteine

A nurse is providing education about dietary modifications to the parent of a school age child who
has glomerulonephritis. Which of the following information should the nurse include in the teaching?
A. Increase the child calcium intake
B. Decrease the Child’s sodium intake
C. Increase the child’s intake of carbohydrates
D. Decrease the child’s fat intake
B. Decrease the Child’s sodium intake

A nurse is providing teaching to the parents of a school-age child newly diagnosed with a seizure
disorder. The nurse should teach the parents to take which of the following actions during a seizure?
A. Minimize movement of the limbs
B. Insert a tongue blade between the teeth
C. Clear the area of hard object
D. Place the child in a prone position
C. Clear the area of hard object

A nurse is assessing an adolescent who has type 1 diabetes mellitus. Which of the following findings is the nurse’s priority?
A. HbA1C 11.5%
B. cholesterol 189 mg/dL
C. Preprandial blood glucose 124 mg/dL
D. Glycosuria
A. HbA1C 11.5%

A nurse is providing anticipatory guidance to a parent of a 1- month-old infant. The nurse should include that it is recommended to start this series of which of the following immunization first?
A. Varicella
B. measles, mumps, rubella
C. Inactivated poliovirus
D. Hepatitis A tetra
C. Inactivated poliovirus

A nurse is reviewing the laboratory report of a toddler who has hemolytic uremic syndrome. Which of the following findings should the nurse expect?
A. Creatinine 0.3 mg/dL
B. Hbg 18 g/dL
C. Urine casts absent
D. BUN 28 mg/dL
D. BUN 28 mg/dL

A nurse is caring for a school-age child who is experiencing a sickle cell crisis. Which of the following actions should the nurse take? (ATI pg. 126)
A. Administer furosemide IV twice per day.
B. Apply warm compresses to the affected areas
C. Decrease the child’s fluid intake
D. Initiate contact precautions.
B. Apply warm compresses to the affected areas

A nurse is assessing a 6-month-old infant who has respiratory syncytial virus. The nurse should immediately report which of the following finding to the provider?
A. Rhinorrhea
B. Tachypnea
C. Pharyngitis
D. Coughing
B. Tachypnea

A nurse is planning to teach an adolescent who is lactose intolerant about dietary guidelines. Which of the following instructions should the nurse include in the teaching?
A. You can drink milk on an empty stomach.
B. You should consume flavored yogurt instead of plain yogurt.
C. You can tolerate plain milk better than chocolate milk.
D. You can replace milk with nondairy source of calcium
D. You can replace milk with nondairy source of calcium

A nurse on a pediatric intensive care unit is caring for a toddler who weighs 12 kg (26.5 Ib) and is postoperative following open heart surgery. Which of the following findings should the nurse report to
the provider?
A. Skin temperature 36C (96.8 F)
B. Pedal and posterior tibial pulses of 2+
C. Urine output of 15 mL in the last 2 hr
D. Drainage from the chest tube of 22 mL in the last hour
C. Urine output of 15 mL in the last 2 hr – urine output should = 1mL/kg/hr =>24mL

A nurse is providing dietary teaching to a parent of a 10-month-old infant who has phenylketonuria. Which of the following responses by the parent indicate an understanding of the teaching?
A. My daughter can’t drink orange juice
B. I will steam carrots and cut them into small pieces for her.”
C. I should ensure that my daughter eats one ounce of meat every day.”
D. I will switch her to whole milk now that she is old enough.”
B. I will steam carrots and cut them into small pieces for her.”

A nurse is providing teaching to the parent of a preschool-age child who has celiac disease. Which of the following instructions should the nurse include?
A. Your child will be on a gluten-free diet for the rest of her life.”
B. Your child will need to follow a low-protein diet temporarily.”
C. You should place your child on a high-fiber diet when she has an exacerbation.”
D. You should replace white flour with wheat flour when preparing meals for your child.”
A. Your child will be on a gluten-free diet for the rest of her life.”

A nurse is administering albuterol by metered dose inhaler for a preschool-age child who is experiencing an asthma exacerbation. Which of the following findings should the nurse report to the
provider?
A. Respiratory rate 24 /min
B. Peak flow rate of 80%
C. Intercoastal retractions
D. Elevated heart rate
C. Intercoastal retractions

A nurse is caring for a school-age child who is 1 hr postoperative following it tonsillectomy. Which of the following actions should the nurse take? (Select all that apply.)
A. Administer an analgesic to the child on a scheduled basis.
B. Observe the child for frequent swallowing
C. Provide cranberry juice to the child.
D. Maintained a child in supine position.
E. Discourage the child from coughing
A. Administer an analgesic to the child on a scheduled basis.
B. Observe the child for frequent swallowing
E. Discourage the child from coughing

A nurse is caring for a school-age child who has heart failure. Which of the following findings should the nurse expect? (select all that apply.)
A. Tachycardia
B. Weight loss
C. Cyanosis
D. Dyspnea
E. Bounding peripheral pulses
A. Tachycardia
D. Dyspnea
E. Bounding peripheral pulses

A nurse in an emergency department is assisting a toddler who has a head injury. Which of the following findings should the nurse report to the provider?
A. Glasgow coma scale score of 15
B. Respiratory rate 25/min
C. Vomiting
D. Negative Babinski reflex
C. Vomiting

A nurse caring for a toddler who is in the terminal stage of neuroblastoma. The parents ask, how can we help our child now? Which of the following responses by the nurse is appropriate?
A. Talk to your child about the meaning of death.”
B. Encourage your child’s friends to visit.”
C. Stay close to your child.”
D. Change your child’s schedule every day.”
C. “Stay close to your child.”

A nurse is preparing to administer cephalexin 25 mg/kg PO to a child who has otitis media and weighs 22 kg (48.5 Ib). Available is Cephalexin solution 250 mg/5 mL how many mL should the
nurse administer? (Round to the nearest whole number. Using a leading Zero if applies. Do not use a trailing zero.)
11 mL

During a well-baby visit, the parent of a 2- week-old newborn tells the nurse, “My baby always keeps her head tilt to the right side. The nurse should further assess which of the following areas?
A.Sternocleidomastoid muscle
B. Posterior fontanel
C. Trapezius muscle
D. Cervical vertebrae
A.Sternocleidomastoid muscle

A nurse is caring for a single mother of a 6-month-old infant. During a well-baby visit, the mother expresses feeling “inexperience” in caring for the baby. The nurse should recommend which of the following community resources?
A. Respite childcare
B. Parent management training
C. Support group for postpartum depression
D. Parent enhancement center
D. Parent enhancement center

A nurse is admitting an infant who has GERD. Which of the following is the priority assessment finding?
A. Regurgitation
B. Wheezing
C. Excessive crying
D. Weight loss
B. Wheezing

A nurse is caring for an infant who has severe dehydration. Which of the following clinical findings should the nurse expect?
A. Capillary refill 3 seconds
B. Rapid respirations
C. Bradycardia
D. Warm extremities.
B. Rapid respirations

A nurse is teaching a group of female adolescents about healthy eating. Which of the following instructions should the nurse include in the teaching?
A. Consume 1,500 to 1,700 calories per day.”
B. Decrease your vitamin D intake once you start to menstruate.”
C. Increase the amount of your dietary iron intake.”
D. Limit your sodium intake to 3,000 grams per day.”
C. ‘Increase the amount of your dietary iron intake.”

A nurse is preparing to administer immunization to a 3-month-old infant. Which of the following is an appropriate action for the nurse to take to deliver atraumatic care?
A. Provide a pacifier coated with an oral sucrose solution prior to the injections.
B. Inject the immunizations into the deltoid muscle
C. Apply eutectic mixture of local anesthetics (EMLA) cream immediately before the injections.
D. Use a 20-gauge needle for the injections.
A. Provide a pacifier coated with an oral sucrose solution prior to the injections.

A nurse is caring for a child who has impetigo contagiosa that developed in the hospital. Which of the following actions should the nurse take?
A. Report the disease to the state health department.
B. Administer amphotericin B IV.
C. Initiate contact isolation precautions.
D. Applying lidocaine ointment topically.
C. Initiate contact isolation precautions.

A nurse is providing discharge teaching to the parents of a school-age child who has cystic fibrosis. Which of the following responses by the parents indicate an understanding of the teaching?
A. I will limit my child’s daily fluid intake.”
B. I will restrict the amount of sodium in my child’s diet.”
C. I will give my child pancreatic enzymes with snacks and meals.”
D .I will prepare low-fat meals with limited protein for my child.”
C. “I will give my child pancreatic enzymes with snacks and meals.”

A nurse is caring for a 4-year-old child who has meningitis and is receiving gentamicin. Which of the following laboratory values should the nurse report to the provider?
A. Creatinine 1.4 mg/dL
B. Creatinine 0.3 mg/dL
C. BUN 6 mg/dL
D. BUN 12 mg/dL
A. Creatinine 1.4 mg/dL

A nurse is providing teaching to the parent of a school-age child who has ADHD and a new prescription for methylphenidate. The nurse should explain that this medication will have which of the following therapeutic effects?
A. Promoting rest
B. Improving appetite
C. Reducing anxiety
D. Increasing focus
D. Increasing focus

A nurse is teaching an adolescent how to manage his cystic fibrosis. which of the following statements by the adolescent indicates an understanding of the teaching?
A. I will take fewer enzymes when I eat high-fiber foods.”
B. I will be excused from physical education classes.”
C. I will limit my calcium intake to prevent kidney stones.”
D. I will increase my intake of vitamin D
D. I will increase my intake of vitamin D

A nurse in a provider’s office is caring for a preschool-age child who might have acute epiglottitis. Which of the following actions should the nurse take?
A. Examine the oral mucosa using a tongue depressor.
B. Obtain a sterile throat culture.
C. Provide humidified oxygen via nasal cannula.
D. Allow the child to sit in a comfortable position.
C. Provide humidified oxygen via nasal cannula.

A nurse is providing teaching to the parents of a child who has impetigo. Which of the following instructions should the nurse include in the teaching?
A. Administer as acyclovir PO two times per day.
B. Soak hairbrushes in boiling water for 10 minutes
C. Apply bactericidal ointment to lesions.
D. Seals soft toys in a plastic bag for 14 days.
C. Apply bactericidal ointment to lesions.

A nurse is preparing to perform a venipuncture to collect a blood sample from an infant. Which of
the following restraints should the nurse plan to use for this procedure?
A. Mummy
B. Mitten
C. Jacket
D. Elbow
A. Mummy

A nurse is reviewing the laboratory report of a school age child who has rheumatic fever. Which of the following laboratory findings should the nurse expect?
A. Decreased BUN
B. Increased antistreptolysin O titer (ASO)
C. Increased immunoglobulin G (IgG)
D. Decreased erythrocyte sedimentation rate (ESR)
B. Increased antistreptolysin O titer (ASO)

A nurses administering an opioid to an adolescent who is in sickle cell crisis. Which statement is
true regarding opioid pain management?
A. Oral opioid doses should be larger than parenteral doses
B. Oral opioids should not be combined with other types of pain relievers.
C. Opioid doses should be titrated until sedation occurs
D. Opioid doses should be used for mild pain
A. Oral opioid doses should be larger than parenteral doses –

A nurse is planning care for an adolescent following repair of Meckel diverticulum. Which of the following actions should the nurse include in the plan of care?
A. Administer total parenteral nutrition.
B. Teach the client about ostomy care.
C. Initiate long-term antibiotic therapy.
D. Maintain an NG tube for decompression.
D. Maintain an NG tube for decompression.

A nurse is preparing to perform peritoneal dialysis for a child who has an elevated serum creatinine
level. After explaining the procedure, which of the following action should the nurse plan to take?
A. Initiate IV access
B. Keep the dialysate refrigerated until time of infusion
C. Check the fistula site for a bruit.
D. Obtain the child’s weight
D. Obtain the child’s weight

A nurse is caring for an adolescent who is one hour postoperative following an appendectomy. Which of the following findings should the nurse report to the provider?
A. Muscle rigidity
B. heart rate 63/min
C. temperature 36.4 C (97.5 F)
D. abdominal pain
A. Muscle rigidity

A nurse is caring for a preschool-age child who is postoperative following a tonsillectomy and is
clearing her throat frequently. Which of the following actions should the nurse take first?
A. Give the child small sips of water.
B. Observe the child’s throat with a flashlight.
C. Administer an Analgesic.
D. Offer the child an ice collar
B. Observe the child’s throat with a flashlight.

A nurse is planning care for a Toddler who has developed oral ulcers in response to chemotherapy. Which of the following actions should the nurse include in the plan of care?
A. Clean the gums with Saline soaked gauze.
B. Administer oral viscous lidocaine.
C. Schedule routine oral care every 8 hr.
D. Moisten the mucosa with lemon glycerin swabs
A. Clean the gums with Saline soaked gauze.

A nurse is planning care for a child immediately following the insertion of a chest tube for
continuous suction with a closed drainage system. Which of the following interventions should the nurse include in the plan of care?
A. Change the chest tube insertion site dressing every 12 hr.
B. Report the presence of tidaling of fluid in the water seal chamber.
C. Ensure continuous bubbling is present in the suction control chamber
D. Record the amount of chest tube drainage every 2 hr.
A. Change the chest tube insertion site dressing every 12 hr.

A nurse is prioritizing care for 4 clients. Which of the following clients should the nurse assess 1st?
A. An adolescent who is in skin traction and report a pain level of 7 on a scale from 0 to 10
B. An adolescent who has sickle cell anemia and slurred speech
C. A toddler who has a new diagnosis of osteomyelitis and is to receive an IV bolus of nafcillin
D. A toddler who has a partial-thickness burn on his right hand and requires a dressing change.
B. An adolescent who has sickle cell anemia and slurred speech – indicates stroke

A nurse is assisting an adolescent who has Cushing’s syndrome. Which of the following findings
should the nurse expect?
A. Cachectic appearance
B. Blood glucose 320 mg/dL
C. Potassium 4.2 mEq/L
D. Advanced bone age
B. Blood glucose 320 mg/dL

A nurse is caring for a preschooler who has a brain tumor. Which of the following findings is the priority for the nurse to report to the provider?
A. Nightmares
B. Pruritus
C. Diplopia
D. hyperactivity
C. Diplopia

A charge nurse is planning care for an infant who has failure to thrive. which of the following
actions should the nurse include in the plan of care?
A. Give the infant fruit juice between feedings
B. Use half-strength formula when feeding the infant.
C. Keep the infant in a visually stimulating environment.
D. Assign consistent nursing staff to care for the infant.
D. Assign consistent nursing staff to care for the infant

A nurse is providing discharge teaching to the parents of an infant who is at risk for sudden infant death syndrome is (SIDS). Which of the following statements by the parents indicates an understanding
of the teaching?
A. I will move my baby stuffed animal to the corner of her crib while she sleeps.”
B. I will dress my baby in lightweight clothing to sleep.”
C. I will have my baby sleep next to me in bed during the night.”
D. I will lay my baby on her side to sleep for naps.”
B. “I will dress my baby in lightweight clothing to sleep.”

A nurse is caring for a child who has acute glomerulonephritis. Which of the following findings
should the nurse expect?
A. Temperature 39 C (102.2 F)
B. Periorbital edema
C. Hypotension
D. Positive urine culture
B. Periorbital edema

A nurse is assessing a 1-month- old infant at a well-child visit. Identify the location the nurse
should stroke to elicit this rooting reflex. (You will find hot spot to select in the artwork below. Select only the hot spot that corresponds to your answer. )
Cheek

A nurse is providing postoperative care for a child following an arterial cardiac catheterization. Which of the following actions should the nurse take?
A. Keep the affected extremity straight for at least 6 hr.
B. Monitor output using an indwelling urinary catheter for the first 24 hr.
C. Remove the child’s pressure dressing after the first 4 hr.
D. Maintain the child’s NPO status for 4 to 6 hr.
A. Keep the affected extremity straight for at least 6 hr.

A nurse in a provider’s office is providing teaching to the parents of a preschooler who has Down
syndrome. Which of the following statements by one of the parents indicate an understanding of the
instructions?
A. We’ll have soft music playing in the background when we teach our son in new skill
B. We’ll explain that it’s best for our son to wait until kindergarten to start going to school
C. we’ll be sure to demonstrate a new skill before expecting our son to perform it .”
D. We’ll focus on our son understanding the principles of a skill rather than mastering it.”
C. “we’ll be sure to demonstrate a new skill before expecting our son to perform it .”

A nurse is teaching a parent of a 10-month-old infant about home safety. Which of the following
instructions should the nurse include in the teaching? (Select all that apply.)
A. Remove labels from containers that contain toxic substances
B. Select a toy chest that has a heavy, hanged lid
C. Place gates at the top and bottom of the stairs.
D. Keep toilet lids in the upright position.
E. Ensure the crib mattress is in the lowest position.
C. Place gates at the top and bottom of the stairs.
E. Ensure the crib mattress is in the lowest position.

A nurse is providing discharge teaching to a parent of a toddler who has a ventriculoperitoneal
shunt. which of the following statements by the parents indicates an understanding of the teaching?
A. My child will need to take prophylactic antibiotics daily until they shunt is removed.”
B. I should call my doctor if my child begins vomiting.”
C. I should pump the shunt at the same time each day.”
D. I should check my child’s heart rate before administering medications.”
B. “I should call my doctor if my child begins vomiting.”

A nurse in a provider’s office is assessing the vital signs of a 2-year-old child at a well-child visit.
Which of the following findings should the nurse report to the provider?
A. Temperature 37.2C (99 F)
B. Respiratory rate 26/min
C. Blood pressure 118/74 mm Hg
D. Pulse rate 98/min
C. Blood pressure 118/74 mm Hg

A nurse is assessing a 3-month-old infant who has diarrhea. Which of the following findings should the nurse expect?
A. Bulging fontanel
B. Decreased heart rate
C. Polyuria – diarrhea indicated dehydration
D. Increased hematocrit
D. Increased hematocrit – diarrhea indicated dehydration

A nurse is preparing to administer imipenem/cilastatin 25 mg/kg to a child who weighs 77 Ib. How
many mg should the nurse plan to administer? (Round the answer to the nearest whole number. Use a leading zero if it applies. Do not use a trailing zero.)
875mg

A nurse is providing teaching to a parent of an infant who has a 1 cm (0.4 in) umbilical hernia.
Which of the following instructions should the nurse include in the teaching?
A. Place a belly band around you baby’s umbilicus during the day.”
B. You should place your baby on her abdomen to sleep at night.”
C. Your baby will need surgery if it doesn’t close by 2 years of age.”
D. The bulge can temporarily enlarge when your baby cries.”
D. “The bulge can temporarily enlarge when your baby cries.”

A nurse is admitting a child who has pertussis. Which of the following transmission-based
precautions should the nurse initiate?
A. Airborne
B. Contact
C. Protective
D. Droplet
D. Droplet

A nurse is assessing a toddler who has a history of lead poisoning. Which of the following actions should the nurse take?
A. Initiate a low-iron diet for lead absorption.
B. Inspect the skin for discoloration.
C. Obtain a stool specimen for lead levels.
D. Perform development testing for delays.
D. Perform development testing for delays.

A nurse is reviewing the medical record of a 24-month-old child who has acute lymphocytic
leukemia. Which of the following actions should the nurse take? (Click on the Exhibit button for
additional information about the client. There are three tabs that contain separate categories of data.)
A. Obtain a rectal temperature every 4 hr.
B. Apply viscous lidocaine to the oral mucosa
C. Place the child in knee-chest position.
D. Initiate bleeding precautions.
D. Initiate bleeding precautions.

A school nurse is assessing a 7-year-old student. The nurse should identify which of the following findings as a potential indicator of physical abuse?
A. Weight in 45th percentile
B. Front deciduous teeth missing
C. Bruising around the wrists
D. Abrasions on the knees
C. Bruising around the wrists

A nurse is providing teaching to the parent of a school-age child who has diabetes mellitus about
managing diabetes during illness. Which of the following statements by the parent indicate an understanding of the teaching?
A. I will monitor my child’s blood glucose levels every 8 hours.
B. I will offer my child 20 grams of carbohydrate every 2 hours.
C. I will withhold my child’s dose of insulin when his appetite is poor
D. I will increase the amount of fluids I offer my child.
D. I will increase the amount of fluids I offer my child.

A nurse is providing discharge teaching to the parents of a toddler who has iron deficiency anemia and new prescription for ferrous sulfate elixir. Which of the following instructions should the nurse include?
A. Don’t allow your child to have orange juice while taking this medication.
B. Administer this medication to your child with a dropper.
C. Give your child this medication with a glass of milk.
D. Stop this medication if you child’s stools are a tarry green color.
B. Administer this medication to your child with a dropper

A nurse is caring for an infant who has tetralogy of Fallot and is having a hypercyanotic episode
after crying. Which of the following interventions should the nurse implement?
A. Initiate continuous positive airway pressure.
B. Provide firm stimulation to the infant’s trunk.
C. Place the infant in the knee-chest position.
D. Perform postural drainage.
C. Place the infant in the knee-chest position

A nurse is providing teaching to an adolescent who has Vulvovaginitis. Which of the following statements should the nurse include in the teaching? This is a trick question. No consensus.
A. Wear a feminine deodorant pad for vaginal drainage.”
B. Wear nylon underwear at night.”
C. Apply scented baby powder to absorb residual moisture.”
D. Apply a warm, moist compress three times per day.”
D. Apply a warm, moist compress three times per day.”

A nurse is providing discharge instructions to the parents of a toddler who has heart failure and a new prescription for digoxin. Which of the following statements indicate an understanding of the
instructions?
A. We will wait to give the medication at the next scheduled time if a dose is missed
B. we will mix the medication with 1 cup of fruit juice for administration
C. We will avoid giving our child water for 1 hour after administrating the medication
D. We will repeat the dose if our child vomits shortly after administration.”
A. We will wait to give the medication at the next scheduled time if a dose is missed

A nurse is planning on in-service for parents of school- age children about the treatment of
pediculosis capitis. Which of the following instructions should the nurse plan to include in the
teaching?
A. Soak the child’s hair brushes in vinegar between uses.
B. Applied medication to the child’s scalp twice daily until the symptoms subside
C. Remove nits from the child’s hair using a fine-tooth comb
D. Discard the child’s nonwashable items.
C. Remove nits from the child’s hair using a fine

A nurse is assessing an adolescent who has infectious mononucleosis. Which of the following findings should the nurse expect?
A. Cervical adenopathy
B. Strawberry tongue
C. Koplik spots
D. Uncontrolled drooling
A. Cervical adenopathy

A nurse in an emergency department is assisting an adolescent who reports inhalation of gasoline.
Which of the following findings should the nurse expect?
A. Ataxia
B. Hypothermia
C. Pinpoint pupils
D. Hyperactive reflexes
A. Ataxia

A nurse is preparing to assess a 4-year-old child’s visual acuity. Which of the following actions
should the nurse plan to take?
A. Position the child 4.6 meters (15 feet) from the chart
B. Use a trumbling E chart for the assessment.
C. Test the child without glasses before testing with glasses.
D. Assess both eyes together first, then each eye separately.
B. Use a trumbling E chart for the assessment.

A nurse in an emergency department is caring for a child following an overdose of acetylsalicylic acid. Which of the following medications should the nurse plan to administer?
A. Phytonadione
B. Midazolam
C. Naloxone
D. Flumazenil
A. Phytonadione – aka Vitamin K

A nurse is providing teaching to the parents of a toddler who is exhibiting negativism during
mealtimes. Which of the following statements by the nurse is appropriate?
A. Tell her she is having her favorite sandwich for lunch.”
B. Ask her if she would like to have her favorite sandwich for lunch.”
C. Ask her if she is ready to eat her sandwich for lunch.”
D. Tell her that she may have a sandwich or soup for lunch.”
D. “Tell her that she may have a sandwich or soup for lunch.”

A nurse in an emergency department is caring for a child who weighs 18 kg (39.7 Ib) and ingested six 500 mg acetaminophen tablets 4 hr ago. Which of the following actions should the nurse take?
A. Prepare to give oral N-acetylcysteine.
B. send a child home on increased fluid intake.
C. Begin hemodialysis within the next 24 hr.
D. Perform gastric lavage with activated charcoal
A. Prepare to give oral N- acetylcysteine

Leave a Comment

Scroll to Top